SlideShare a Scribd company logo
1
Real-WorldBoards
Question 1 of 82
For about 2 weeks, initially with nasal congestion, rhinorrhea and a sore throat,
now with a steady cough productive of yellow sputum. She feels better except for
the cough, which is impairing her functioning at work. Her temperatures (taken at
home) have all been under 101 F.
You notice she seems a bit under the weather but is breathing normally; you take
her radial pulse, which is 89. She asks you what she should buy for her cough.
What's the best recommendation for her, based on available evidence?
A. Cetirizine for 7 days
B. Brompheniramine with pseudoephedrine, and naprosyn, until symptoms subside
C. Azithromycin for 5 days
D. Chicken soup
E. Schedule an office visit with chest films
You answered.
We feelthat B and D are the best answers.
Acute cough (< 3 weeks) is believed to be most often due to the common cold.
Cough can be persistent and bothersome, justifying treatment. One randomized
trial showed improvement in cough with the use of brompheniramine (a first
generation antihistamine) with pseudoephedrine. Naprosyn bid also reduced cough
(and other cold symptoms)in a randomized trial. These two therapies are "A"
2
recommendations by the ACCP for patients with bothersome cough due to the
common cold.
Loratadine with pseudoephedrine was no better than placebo in another study;
other studies also show that newer-generation nonsedating antihistamines are not
effective on cold symptoms.
A systematic review concluded that in fact, no over-the-counter remedies are
effective for cough from the common cold; however, it lumped together new
antihistamines (ineffective) and older ones (possibly effective) and concluded
antihistamines are not effective. It also doubted whether the benefits seen in
positive trials were clinically significant. Reassurance seems a reasonable option,
too.
Office visits, chest films, and antibiotics are not needed for healthy people with
colds. Fever, immune suppression, tachycardia, or tachypnea might prompt a chest
X-ray.
Also: Acute bacterial sinusitis can present with postnasal drip cough and upper
respiratory tract infection, and be indistinguishable from the common cold (even
on sinus imaging). Cough that persists > 3 weeks should be evaluated as chronic
cough. Pertussis causes persistent cough, is underdiagnosed, and should be treated
with macrolide antibiotics. The CDC recommends pertussis boostervaccination for
all adults.
Question 2 of 82 (score?)
This 78 year old white woman has been coughing for "a good while"; you estimate
at least many months by her history. She is persistently fatigued, has lost 5 pounds,
but has no fevers or night sweats. She is a never-smoker. She sometimes coughs up
blood-streaked sputum. She lives in a high-rise retirement home. Other cuts of her
chest CT are similar to the one below, correlating well with the findings on chest
X-ray.
3
4
The primary problem is likely:
A. An infection she caught from the environment
B. An infection she caught from someone else
C. Reactivation tuberculosis
D. Bronchioloalveolar carcinoma
E. A chronic process resulting from repeated past infections
We feelthat A is the best answer.
This is a mycobacterium avium complex infection, with Lady Windemere
syndrome (bronchiectasis and multiple small nodules often involving the right
middle lobe or lingula in nonsmoking elderly women, often with chest wall or
skeletal deformities). The other options are possible but the pattern on imaging
along with the history are most suggestive of MAC, an endemic organism inhaled
5
from soil or water. Her bronchiectasis could perpetuate her cough and hemoptysis;
however, she also has many nodules that are indicative of ongoing infection as the
primary process.Nontuberculous mycobacteria (NTM) also include M. kansasii,
rapidly growing mycobacteria (RGM), and numerous less common species.
Nonpathogenic airway colonization by these organisms is frequently incidentally
discovered and must be differentiated from infection. NTM infection commonly
causes a TB-like pattern (upper lobe cavitary disease); hypersensitivity
pneumonitis (ground glass and centrilobular nodules, sometimes with a mosaic
pattern), or the pattern seen here (nodular bronchiectasis with or without
fibrocavitary disease), along with low-grade nonspecific symptoms usually
including cough and fatigue.
Diagnosis requires compatible imaging and clinical findings, along with 2 positive
sputum samples (24+ hrs apart) or 1 positive sample from bronchoscopic lavage or
biopsy. This unfortunate woman has required more than 3 years of multi drug
therapy, with repeated medication switches for adverse effects including optic
neuropathy while on ethambutol--still, her sputum remains positive for MAC.
Unfortunately, her situation is not atypical for people with this indolent, difficult-
to-treat infection. National Jewish in Denver are the premier referral / consultation
center for these cases.
Question 3 of 82 (score?)
You receive an email from a colleague asking for a "curbsideconsult." You haven't
heard the story or your colleague's question yet, but to frame your thinking, you
quickly glance at the images she's attached to the email.
6
7
8
Based on these images, what's the most likely clinical scenario?
A. A 50-year old man with a mass in the head of the pancreas.
B. A 66-year old man with progressive dyspnea on exertion, leg edema, and
longstanding hypertension.
C. A 48-year old woman treated for malignant melanoma 2 years ago.
D. A 24-year old woman with HIV-AIDS, 4 weeks of dyspnea and dry cough,
SaO2 of 88% on ambient air, non-adherent with Bactrim prophylaxis.
E. A 60-year old chicken farmer with progressive dyspnea and a positive
hypersensitivity panel.
We feelthat A is the best answer.
The chest X-ray shows reticulonodular opacities, a nonspecific abnormal finding.
The axial CT cuts show:
 Small nodules in a lymphangitic pattern, i.e., touching the pleura/fissures
 Pleural effusions with an irregular, nodular contour
9
 Irregular thickening of the interlobular septa
 Peribronchovascular thickening
All these findings are consistent with this patient's diagnosis of lymphangitic
carcinomatosis(LC): diffuse metastatic spread of cancer to the lungs, where it
diffusely infiltrates and obstructs lymphatic channels. 80% of cases are due to
adenocarcinomas, most often from a primary cancer of the breast, lung, or colon.
This case of LC was from a primary pancreatic adenocarcinoma.
Chest films are usually normal in lymphangitic carcinomatosis.
This patient's imaging studies are also consistent with:
 Sarcoidosis (would expect hilar lymphadenopathy)
 Lymphoma
 Silicosis
 Coal worker's pneumoconiosis
Regarding the other answer choices, all but metastatic melanoma would likely
include reticular opacities on chest X-ray. Their patterns on CT imaging would
most likely include:
 Cardiogenic pulmonary edema (CHF): Bilateral/diffuse interlobular and
intralobular septal thickening, ground glass opacities, pleural effusions.
Centrilobular nodules (ill-defined/hazy) may be seen, surrounded by
ground glass opacity.
 Metastatic melanoma: Nodules/masses, usually multiple, most often >1-2
cm, in a random pattern affecting one or both lungs. Diffuse lymphangitic
involvement would be highly unusual.
 Pneumocystis pneumonia: Bilateral/diffuse ground glass opacities (90%);
nodules are rare.
 Hypersensitivity pneumonitis: Bilateral/diffuse centrilobular nodules,
surrounded by ground glass opacities, often with tree-in-bud opacities. Later
in the illness, fibrosis (traction bronchiectasis, honeycombing) is present.
Nodule mini-primer:Centrilobular nodules appear at the center of the secondary
pulmonary lobules; they represent engorgement of the pulmonary arteriole and/or
occlusion of the centrilobular bronchiole. Centrilobular nodules do not touch the
pleural surfaces or fissures. (Lymphangitic nodules and random nodules may touch
the fissures/pleural surfaces.) Lymphangitic nodules are accompanied by irregular
thickening of septae and bronchovascular bundles; they occurfrom lymphangitic
10
spread of tumor, sarcoid, or another inflammatory process. Randomnodules result
from hematogenous spread of tumor or disseminated infection (tuberculosis, fungal
infection) and occurwithout the associated findings of lymphangitically-spread
nodules.
Question 4 of 82 (score?)
Your colleague in the ED calls you. He has a 55-year-old former nurse there, sent
by her PCP for leg swelling.
He ordered lower extremity ultrasound; the left common femoral vein image is
shown. The image on your left is with the probe resting on the skin; the image on
the right is with compressionof the vein by the ultrasound probe. She has edema in
the left leg. She has no risk factors for or history of DVT. Vital signs are normal. A
radiologist will be available to interpret the images in a few hours. After your
history and exam reveal nothing else, the ED doc asks what you want to do.
Of the listed options, what is the best recommendation you could make?
A. Schedule placement of an IVC filter while awaiting the final radiology read.
B. Admit; start warfarin and unfractionated heparin; recommend warfarin for 3
months.
C. Start enoxaparin and warfarin, then discharge; recommend warfarin indefinitely.
11
D. Get dedicated Doppler ultrasound of the leg veins.
E. Pulmonary angiogram.
We feelthat C is the best answer.
This is a proximal, unprovoked, asymptomatic DVT. The vein should compress
completely; this one does not, and has an echogenic area in the lumen (it should be
homogenously jet-black inside).
Generally speaking, recurrence rate after unprovoked DVT is similar between
people with and without identifiable hypercoagulable states. ACCP
recommendations for people with unprovoked proximal DVT without PE are that
anticoagulation be continued "long-term" (i.e., indefinitely), if the patient is
amenable after a risk/benefit discussion(Grade 1A, "Antithrombotic Therapy for
VTE Disease," Section 2.1.2). ACCP also recommends warfarin be begun the same
day as enoxaparin.
There is growing acceptance of treating asymptomatic DVT on an outpatient basis
"if possible" (Grade 1C ACCP recommendation), which we take to mean the
patient is stable, can self-inject enoxaparin, and close follow-up can be arranged
for INR checks.
Question 5 of 82 (score?)
Your internal medicine colleague asks you about a patient she is about to discharge
home after a hospitalization for a COPD exacerbation. The patient, Mr. M., takes a
beta-blocker for hypertension. Mr. M. does not have a diagnosis of coronary artery
disease. Your colleague is considering stopping the beta-blocker to avoid any
contribution to future COPD exacerbations, but wants your opinion first.
What do you recommend?
A. Stop the beta blocker.
B. Continue the beta blocker.
C. Stop the beta blocker; order a stress test.
D. Continue the beta blocker; order an echocardiogram.
We feelthat B is the best answer.
Cardioselective beta-blockers are safe in patients with COPD, and may in fact be
beneficial, mounting evidence suggests. A cohortstudy published in 2008 among
12
>3,000 people with cardiovascular disease suggested beta blockade is not
associated with a reduced FEV1 or increased exacerbations. In an observational
database study of almost 6,000 outpatients with COPDpublished in 2011, beta
blocker use was associated with reduced mortality and fewer COPDexacerbations.
Cardioselective beta-blockers are likely beneficial for patients with mild or
moderate asthma, as well, although that data is less conclusive than for COPD.
Early case reports of beta blockers precipitating asthma exacerbations were in
patients taking noncardioselective beta blockers (e.g., propranolol).
Clinical Takeaway:Beta-blockers may be prescribed without restriction to
people with COPD, if there is no contraindication.
Question 6 of 82 (score?)
A 46-year-old woman with persistent asthma presents for her routine 3 month visit.
She notes her symptoms have been worse for the past few weeks. She is using
albuterol daily, and having cough and wheezing a few nights a week. She has
stopped her daily walk for exercise, due to difficulty breathing.
Her vital signs are normal. On exam, she is breathing comfortably but has
wheezing diffusely.
She is taking fluticasone 220 mcg twice daily (medium dose) and albuterol 3 times
daily. She demonstrates her inhaler technique, which is correct.
What should be the next step?
A. Schedule spirometry for next week to guide step-up therapy.
B. Increase the doseof fluticasone to 440 mcg b.i.d.
C. Provide prednisone 40 mg for 5 days and see again next week.
D. Add a long-acting beta agonist.
E. Add a leukotriene receptor antagonist..
We agree that B, C, D, and E are all reasonable answers.
The patient needs step-up therapy for uncontrolled asthma. Traditionally that
would have meant adding a long-acting beta agonist, but the FDA's 2010 warnings
13
about long-acting beta agonists for asthma have introduced uncertainty (or call it
flexibility) into clinical decision making for uncontrolled asthma.
The National Asthma Education and Prevention Program (NAEPP guidelines) for
2007 would consider her asthma to be either:
 "Notwell controlled": Step-up by 1 step; see again in 2-6 weeks; use
alternative treatments if side effects develop.
 "Very poorly controlled":Consider short course of oral steroids; step-up
1-2 steps;see again in 2 weeks; use alternative agents if side effects develop.
Per NAEPP guidelines, step-up therapy would definitely include adding a long-
acting beta agonist, and 2 steps up would also include increasing to high doseICS
(440 or 500 mcg fluticasone twice daily). NAEPP-recommended alternatives to a
LABA include a leukotriene receptor antagonist, Zileuton, theophylline, and at
higher steps, omalizumab for those with allergies.
Since February 2010, FDA black-box warnings for LABAs in asthma have created
uncertainty around these guidelines. The most current language at FDA.gov (June
2010) would supportLABA use in this patient (inadequate control on medium-
doseICS). Given the confusion at present, though, it would be hard to argue with a
physician who choseto use an alternative agent in place of a LABA, or increase
the inhaled corticosteroid dosewithout adding a LABA.
Under direction by the FDA, major pharma companies are launching 5 large
clinical trials to answer the question of LABA safety for asthma; results are
expected in ~2018 or so.
Delaying step-up therapy to obtain spirometry does not seem appropriate.
Spirometry can help place a patient in a category on the NAEPP algorithm;
however, regardless of the test result, step-up therapy is indicated and should be
started now.
Question 7 of 82 (score?)
John, 33, is referred to you by his primary doctorfor bronchiectasis. John has had
recurrent respiratory infections since graduating high school. He takes inhaled
albuterol as needed, which in practice is never, becausehe does not perceive a
benefit. His symptoms of episodic cough, dyspnea, and infections wax and wane
unpredictably, but are progressing overall. Exam and vitals reveal distant breath
sounds with some rhonchi, and are otherwise normal. A thorough occupational /
14
inhalational history is negative. As you consider beginning an extensive workup
for unexplained bronchiectasis, you examine the test results below.
Question 8 of 82 (score?)
A 74-year-old man with severe, end-stage COPD(FEV1 20% predicted) suffers
from breathlessness that is so severe, any activity (even standing from a chair)
makes him feel like "I'm gonna die." He is on maximal inhaler therapies and
supplemental oxygen. He has tried pulmonary rehabilitation but he could not
tolerate it.
To reduce this patient's feelings of severe breathlessness, which intervention has
the strongest evidence for effectiveness?
A. Nebulized morphine sulfate.
B. Oral morphine sulfate.
C. Using a fan to blow air on his face.
D. Biofeedback therapy.
We feelthat B is the best answer.
Several systematic reviews and meta-analyses of randomized trials supportthe use
of opioid use to relieve severe dyspnea in people with end-stage COPD. Many
other therapies to relieve dyspnea in COPDhave been tested in randomized trials.
Positive results were found from chest wall vibration and neuroelectrical muscle
stimulation. Nebulized morphine also did not show an effect in randomized trials.
Acupuncture, music therapy, use of a fan, and biofeedback/relaxation showed
inconsistent or no benefits. The lack of conclusive evidence in randomized trials
does not mean these nonpharmacologic therapies shouldn't be tried, as they may
work in individual patients, and a placebo or psychological effect may be
beneficial to maximize comfort in a palliative treatment plan, especially with
therapies that are safe and simple.
15
16
17
What's the diagnosis?
A. Williams-Campbell syndrome.
B. Allergic bronchopulmonary aspergillosis.
C. Mounier-Kuhn syndrome.
18
D. Cystic fibrosis.
E. Marfan syndrome.
We feelthat C is the best answer.
Mounier-Kuhn syndrome, or congenital tracheobronchomegaly, is a rare inherited
disorder of cartilage formation resulting in enlargement of the C-rings in the
segmental bronchi and trachea. More distal bronchial structures are normal. (This
is in contradistinction to Williams-Campbell syndrome, in which central airways
are normal but distal airways are dilated.) The diagnosis is readily made on a CT
scan showing grossly enlarged central airways, especially with compatible
symptoms.
Clinical presentation is usually with recurrent lower respiratory infections, which
may begin in childhood, or not until young adulthood. Diagnosis was delayed until
age ~75 in a few cases. Productive cough, dyspnea, and poorclearance of
secretions are common symptoms.
Prognosis varies widely and is impossible to accurately predict. Progressive
bronchiectasis (perhaps due to repeated infections), emphysema, and pulmonary
fibrosis are possible. Mild, stable cases, as well as those progressing to respiratory
failure and death, have been reported.
Due to the condition's rarity, no evidence is available to guide management, but
airway hygiene has been recommended: postural drainage and consideration of
other airway clearance techniques (chest physiotherapy, flutter valves,
bronchodilators, dornasealpha, etc). Positive-airway pressure and airway stenting
have been proposed. Surgery (posterior membranous tracheobronchoplasty) may
be helpful in certain severely affected patients, but is usually not technically
feasible.
The cartilage defect in Marfan syndrome can cause tracheobronchomalacia, and
either Marfan, cystic fibrosis, or ABPA can cause bronchiectasis with cystic
degeneration of the lungs, but none of these cause tracheobronchomegaly.
19
Question 8 of 82 (score?)
A 74-year-old man with severe, end-stage COPD(FEV1 20% predicted) suffers
from breathlessness that is so severe, any activity (even standing from a chair)
makes him feel like "I'm gonna die." He is on maximal inhaler therapies and
supplemental oxygen. He has tried pulmonary rehabilitation but he could not
tolerate it.
To reduce this patient's feelings of severe breathlessness, which intervention has
the strongest evidence for effectiveness?
A. Nebulized morphine sulfate.
B. Oral morphine sulfate.
C. Using a fan to blow air on his face.
D. Biofeedback therapy.
We feelthat B is the best answer.
Several systematic reviews and meta-analyses of randomized trials supportthe use
of opioid use to relieve severe dyspnea in people with end-stage COPD. Many
other therapies to relieve dyspnea in COPDhave been tested in randomized trials.
Positive results were found from chest wall vibration and neuroelectrical muscle
stimulation. Nebulized morphine also did not show an effect in randomized trials.
Acupuncture, music therapy, use of a fan, and biofeedback/relaxation showed
inconsistent or no benefits. The lack of conclusive evidence in randomized trials
does not mean these nonpharmacologic therapies shouldn't be tried, as they may
work in individual patients, and a placebo or psychological effect may be
beneficial to maximize comfort in a palliative treatment plan, especially with
therapies that are safe and simple.
Question 9 of 82 (score?)
This 49 year old U.S. citizen originally from Trinidad is referred to you for
evaluation by her oncologist. She had the below chest X-ray after a mammogram
suggested abnormal lung parenchyma. Breast biopsy showed ductal carcinoma in
situ. She is to have lumpectomy soon. She has been mildly short of breath since her
20s, which has insidiously gotten worse, although she still works full-time and
takes care of 3 children. She also has a chronic dry cough worse on deep
20
inspiration. She has never sought evaluation for her symptoms and has never had
prior chest imaging. In your office, she walks 1000 feet in 6 minutes, with
desaturation from 100% on ambient air to 85% on ambulation with mild dyspnea.
A TB skin test is placed. An HIV test was negative last year.
PFTs:TLC 70% predicted (below lower limit of normal), FEV1/FVC ratio 0.65,
FEV1 2.1L (below lower limit of normal) DLCO 40% predicted, DLCO/VA 80%
predicted.
21
22
Without any other information, her most likely diagnosis is:
A. Idiopathic pulmonary fibrosis
B. Sarcoidosis
C. Tuberculosis
D. Lymphangitic spread of breast cancer
We feelthat B is the best answer.
23
Sarcoidosis is more prevalent in people of African descent, and can present at a
young age and progress insidiously. This woman likely has diffuse lung fibrosis
without hilar lymphadenopathy, so-called stage IV sarcoid. The term "stage"
misleadingly implies a predictable pattern of linear disease progression; in fact, the
radiographic pattern (stage) correlates neither with chronicity nor with changes in
pulmonary function.
Although patients usually have restrictive lung disease at presentation, as many as
50% also have obstructive disease. Bronchodilator responsiveness is not
uncommon. Spirometry returns to normal in 80% of patients within 2 years (with
about 67% of patients achieving a complete remission of sarcoid within 10 years).
Sarcoidosis is a diagnosis of exclusion, with a minimum requirement of a biopsy
demonstrating noncaseating granulomas (except in certain patients, such as those
with Lofgren's syndrome, Heerfordt's syndrome, or asymptomatic, incidentally
discovered hilar lymphadenopathy). At that point, other granulomatous diseases
must be excluded. This patient had a noncaseating granuloma on breast biopsy --
which can also be seen in breast cancer and which therefore still left some
diagnostic uncertainty. Her symptoms and imaging have been stable over years
since her first visit; she remained in remission from breast cancer after
lumpectomy and radiation.
Miliary TB would be expected to producea more nodular pattern on chest films, as
opposedto this linear or reticular pattern. In situ cancer would not likely have
spread lymphangitically. IPF usually preferentially affects the peripheral and basal
lungs. The small nodule in the right lung baseis probably of no significance.
Question 10 of 82 (score?)
A 28-year-old medical student approaches you in the hospital hallway. She says
with dismay, "My PPD is positive."
She holds out her arm, which shows 12 mm of induration marked off by an
examiner. You slide the edge of your pen tip along her skin until you meet
resistance on each side. You get the same margins as the examiner's.
The student says, "I just got a PPD 3 weeks ago that was negative - I know it was.
But I got busy and never had it checked."
She has no known exposures to tuberculosis, and no respiratory or constitutional
symptoms. She had a chest X-ray this morning which was normal.
24
She had a negative TB skin test last year when she first came to the US from
Vietnam (a TB-endemic country), where she grew up. She did get BCG vaccine
there as a child.
You refer her to your hospital's occupational health department for further advice
and management.
Assuming the skin test 3 weeks ago was truly negative, and the chest X-ray this
morning was normal, what would be the best recommendation?
A. Repeat skin test next year; this was a boosterreaction.
B. Prescribe isoniazid daily for 9 months.
C. Repeat skin testing in one month.
D. Collect serum for interferon-gamma release assay.
E. Repeat skin test next year; this was a false-positive from BCG vaccine.
We agree that B is the best answer.
The student has a positive tuberculin skin test (TST) confirming latent
tuberculosis infection (LTBI), probably occurring years ago in her home country.
(Cutoff for TST positivity is 10 mm in health care workers; other cutoffs are here.)
She did have a boosterreaction. Boosterreactions are initial false-negatives that
become true-positives. They occurwhen a previously-infected personloses
hypersensitivity to PPD antigen over time, resulting in a negative TB skin test.
However, the injected PPD re-stimulates the immune system, and a TB skin test
repeated 1-3 weeks later will induce a positive induration reaction. Because of this,
the gold standard for TB skin testing is actually to place 2 PPDs 1-3 weeks apart; if
either is positive, the person is considered to have LTBI and treatment should be
considered.
Treatment for LTBI can reduce the lifetime risk of reactivation tuberculosis by
60-90%:
 First-line therapy is 9 months of daily isoniazid (5mg/kg, max 300 mg).
 Six months of INH is also acceptable and supported by randomized trial
data.
 Other acceptable regimens include twice-weekly INH 300 mg for 6 or 9
months, or daily rifampin for 4 to 6 months. Data is less robust for these.
25
Both INH and rifampin have potential toxicity, which must be balanced against the
risk of reactivation tuberculosis. There is a 1 in 1000 risk of hepatitis while taking
INH for alcohol-abstinent people without preexisting liver disease; asymptomatic
transaminitis is common. With monthly monitoring, intermittent dosing, and
provision of pyridoxine, INH can be given safely to most patients.
Without treatment, the lifetime risk of developing reactivationtuberculosis in
HIV-negative people is ~5-10%, with most of that risk in the first 2 years
following infection.
By CDC guidelines, the other answer choices are not recommended:
 BCG-vaccinated people should be tested and treated for LTBI the same as
everyone else.
 Interferon-gamma release assay and TST should not be routinely performed
together. In this case, the patient was tested with the gold standard two-step
approach(although accidentally) and should be considered to have LTBI;
confirmatory testing isn't needed.
 Her skin test would be expected to still be positive in one month. Once a TB
skin test is positive once in life, it will always be positive (or falsely
negative). There is no way to detect re-infection with tuberculosis with skin
testing.
Question 11 of 82 (score?)
During 25 days in your ICU, Mr. F has survived postoperative sepsis after a ventral
hernia repair, and ARDS with prolonged respiratory failure with a tracheostomy.
He still requires 20 mmHg of pressure supportventilation overnight, but tolerates
trach collar during the days. A local long-term acute care hospital is eager to take
over his care, and his transfer paperwork is ready.
His wife, Wanda, wants to know what to expect now. Mr F is too weak and
confused to communicate meaningfully, but she thinks her husband would want to
go on fighting if he had a good chance to survive and get back to a place where he
could at least live at home with assistance.
According to published literature, what are Mr F's chances of being alive in one
year?
26
A. 75%
B. 50%
C. 25%
D. 10%
E. It's impossible to predict from the information provided.
We agree that B and C are the best answers.
Chronic critical illness is an epiphenomenon resulting both from advances in
intensive care and the financial success (and related proliferation) of long-term
acute care (LTAC) hospitals. Generally defined as respiratory failure lasting
weeks, chronic critical illness affects more than 100,000 people in the U.S.
(estimated), and costs more than $20 billion annually.
What happens to your patient after you send him or her to an LTAC?
- Most are not freed from mechanical ventilation (reported weaning rates: 30 -
50%); if success is achieved, it's almost always within 60 days.
- More than 40% are readmitted to the hospital within a year.
- At the end of one year, fewer than half will be alive (32 - 52%).
- Fewer than 12% will be alive and independent 1 year after their acute illness.
Surveys of patients' families show that most have no idea of these grim odds. It's
uncertain whether that's because physicians fail to effectively communicate the
reality of the situation, or whether it's due to "selective hearing" by emotionally
overwhelmed families desperate for hope. Although it's true that we can't predict
with perfect accuracy what will happen to any individual patient, in the absence of
clear reasons to be optimistic it might not be appropriate to say things like "He just
needs more time" or "We just can't predict these things," since the data show that
we usually can (at least in terms of predicting functional independence after LTAC
admission, which is an unlikely outcome
27
What is the bestintravenous medication infusion to provide next?
A. Lidocaine.
B. Esmolol.
C. Heparin.
D. Diltiazem.
We feelthat C is the best answer.
After successfulemergent electrocardioversion of atrial fibrillation, heparin
infusion is appropriate if there is no contraindication. It is not known how long the
patient was in a-fib before, and a dangerous clot may be present in the left atrium /
left atrial appendage, increasing the risk for stroke. Cardiology consultation should
also be sought.
Diltiazem and beta blocker infusions are not necessary since she is in normal sinus
rhythm. Lidocaine is an alternate agent to amiodarone for ventricular tachycardia
or fibrillation.
Question 12 of 82 (score?)
During 25 days in your ICU, Mr. F has survived postoperative sepsis after a ventral
hernia repair, and ARDS with prolonged respiratory failure with a tracheostomy.
He still requires 20 mmHg of pressure supportventilation overnight, but tolerates
trach collar during the days. A local long-term acute care hospital is eager to take
over his care, and his transfer paperwork is ready.
28
His wife, Wanda, wants to know what to expect now. Mr F is too weak and
confused to communicate meaningfully, but she thinks her husband would want to
go on fighting if he had a good chance to survive and get back to a place where he
could at least live at home with assistance.
According to published literature, what are Mr F's chances of being alive in one
year?
A. 75%
B. 50%
C. 25%
D. 10%
E. It's impossible to predict from the information provided.
We agree that B and C are the best answers.
Chronic critical illness is an epiphenomenon resulting both from advances in
intensive care and the financial success (and related proliferation) of long-term
acute care (LTAC) hospitals. Generally defined as respiratory failure lasting
weeks, chronic critical illness affects more than 100,000 people in the U.S.
(estimated), and costs more than $20 billion annually.
What happens to your patient after you send him or her to an LTAC?
- Most are not freed from mechanical ventilation (reported weaning rates: 30 -
50%); if success is achieved, it's almost always within 60 days.
- More than 40% are readmitted to the hospital within a year.
- At the end of one year, fewer than half will be alive (32 - 52%).
- Fewer than 12% will be alive and independent 1 year after their acute illness.
Surveys of patients' families show that most have no idea of these grim odds. It's
uncertain whether that's because physicians fail to effectively communicate the
reality of the situation, or whether it's due to "selective hearing" by emotionally
overwhelmed families desperate for hope. Although it's true that we can't predict
with perfect accuracy what will happen to any individual patient, in the absence of
clear reasons to be optimistic it might not be appropriate to say things like "He just
needs more time" or "We just can't predict these things," since the data show that
29
we usually can (at least in terms of predicting functional independence after LTAC
admission, which is an unlikely outcome
Question 13 of 82 (score?)
John, 44, comes to see you in your clinic, complaining of worsening exertional
dyspnea. He smoked a half a pack a day for 5 years, quitting 25 years ago. He
recently quit his construction job becausehe couldn't unload sacks of concrete or
walk the site quickly enough to keep up.
PFTs:FEV1 1.1 L (35% predicted); FEV1/FVC ratio 0.45; DLCO 35% predicted.
Vitals: HR 90, RR 18, BP 125/85, SaO2 94% on ambient air, 92% with
ambulation.
He saw his primary doctora year ago, who ordered a chest X-ray and chest CT; he
hands you a CD with some cuts of the chest CT, which you review.
30
31
What approachto testing is most likely to yield a correct diagnosis?
A. Repeat PFTs
B. Serum antiprotease testing
C. Bronchoscopywith biopsies
D. Surgical lung biopsy
E. Thorough occupational history
We agree that B is the best answer.
This patient has severe alpha-1 antitrypsin deficiency (A1ATD). Severe A1ATD
is mistakenly believed to be rare; actually, 60,000 to 100,000 Americans are
estimated to have the PI*ZZ genotype associated with severe deficiency (<50
mg/dL or 11 micromoles/L; >20 micromoles/L is normal). Most are of northern /
western European descent and are undiagnosed. Features that should prompt
consideration of A1ATD include:
 Emphysema before age 45
 Absence of smoking history or other exposures (e.g., organic dust)
 Emphysema with basilar lucency
 Unexplained liver disease
 Family history of emphysema, liver disease, or panniculitis
 Necrotizing panniculitis (rare, even in A1ATD patients)
32
The manifestations of A1ATD are complex and difficult to predict:
 Many nonsmokers with PI*ZZ never develop liver or lung disease and seem
to live a near-normal lifespan.
 Smokers with severe A1ATD have wide variation in the rate of FEV1
decline and development of emphysema.
 Many patients with A1ATD have an asthma-like phenotype with
bronchodilator-reversible obstruction; these patients seem to be at greater
risk for FEV1 decline (even if nonsmokers), and ATS recommends treating
them "aggressively" with inhaled steroids and bronchodilators.
 Liver disease is more common in childhood, but as many as 30-40% of
patients aged >50 with A1ATD may have cirrhosis or carcinoma of the liver.
Overall, prognosis in individual patients can't be predicted (except for those with
already severe disease who smoke), becauseof the heterogeneity of the disease and
the lack of prospective longitudinal studies.
Patients with PI*MZand PI*SZ seem to be at increased risk for COPDif they
smoke, but not markedly so if they do not.
Somewhat surprisingly, the ATS recommends testing virtually all Americans with
COPDfor A1ATD. (Also, anyone with unexplained liver disease, necrotizing
panniculitis, asymptomatic obstruction on PFTs, and all siblings of A1ATD
patients.) PI*MM (in 95% of U.S. population) ensures levels >20 micromoles/L
and normal function.
Nonrandomized trials suggest that A1AT supplementation may slow FEV1
decline and improve survival in severe A1ATD. Patients with lower FEV1 (31-
65% predicted) seemed to have a greater benefit. ATS guidelines are vague and
acknowledge the relatively weak evidence, but suggest A1AT augmentation is
appropriate for anyone with severe deficiency (<11 micromoles/L), and those with
A1ATD (<20 micromoles/L) and either of these criteria:
 FEV1 30-65% predicted (postbronchodilator);
 Rapid decline of lung function (FEV1 decline >120mL/yr), regardless of
initial FEV1.
From observational data, the risk of anaphylaxis may be ~1% per patient over ~6
years of A1AT infusions. Patients should probably be given an Epi-Pen and taught
to use it.
33
This patient's CT scanshows panacinar emphysema, with bullous disease worse at
the bases. Panacinar disease can merge with severe centrilobular emphysema to
create a nonspecific CT scan.
Question 14 of 82 (score?)
A 33 year old woman returns to see you for her third visit for significant
progressive dyspnea. She has no other medical conditions, and normal imaging
except for an enlarged pulmonary artery on CT scan. Her echocardiogram
suggested severe pulmonary hypertension (with normal LV function) on her last
visit, so you referred her for right heart catheterization. The report is:
Initial values
After 25 ppm NO
inhaled
Central venous pressure 8 mm Hg 9 mm Hg
Pulmonary artery
pressure
70/39 (mean 47 mm
Hg)
63/36 (mean 43 mm Hg)
PAOP ("wedge") 12 mm Hg 13 mm Hg
Cardiac index 2.8 L/min/m2 2.9 L/min/m2
She's also had a negative V/Q scan, HIV test, sleep study, echocardiographic
bubble study, liver and hepatitis panel, high-resolution CT scan, urine drug screen,
and autoimmune labs.
She is at normal weight and did regular aerobic exercise before, but now cannot
walk 100 feet without stopping for dyspnea. Her oxygen saturations are normal
with exertion. Jugular venous pressure is 11 cm. She has pitting edema over both
tibias.
What's the worstadditional therapeutic recommendation for her?
34
A. Bosentan
B. Sildenafil
C. Epoprostenol
D. Nifedipine
E. Warfarin
We agree that D is the bestanswer.
Idiopathic pulmonary arterial hypertension is a diagnosis of exclusion, arrived at
after an extensive workup as outlined in guidelines and described above. All
patients must undergo right heart catheterization before initiating treatment for
any form of pulmonary arterial hypertension.
A small percentage of patients are "vasoreactive" to adenosine or nitric oxide: they
have a decrease in mean pulmonary artery pressure of >= 10 and to below 40 mm
Hg when these agents are administered during RHC. These patients have an
excellent prognosis as long as they take a vasodilating calcium channel blocker,
such as nifedipine.
Treatment for non-vasoreactive IPAH (as this patient has) may include oral
endothelin receptorantagonists (bosentan), oral phosphodiesteraseinhibitors
(sildenafil), or IV/inhaled prostanoids (epoprostenol, treprostinil, iloprost). U.S.
guidelines recommend the more costly and inconvenient prostanoids for patients at
"high risk" (rapid progression; RV dysfunction or RA pressure > 20 mm Hg; WHO
functional class IV; high BNP; <300 m 6-minute walk).
The 1A recommendations of the European Society of Cardiology are as follows:
Evidence
level
WHO class II WHO class III
WHO class
IV
I - A
Ambrisentan,
bosentan, sildenafil
Ambrisentan, bosentan, sitaxetan,
sildenafil, eprostenolIV, iloprost
inhaled
Epoprostenol
IV
35
Diuretics should be prescribed when volume overload is present. Based on
observational data of improved survival in patients with IPAH, warfarin is
generally indicated as well. In those with an indwelling catheter for IV therapy,
warfarin is believed to be additionally helpful in preventing catheter thrombosis.
Atrial septostomyand lung (or heart-lung) transplantation are options when PAH
medical therapies fail.
Question 15 of 82 (score?)
A 19-year old man presents to the ED with the suddenonset of mild dyspnea and
chest pain. A chest X-ray reveals a right-sided pneumothorax, with 4.2 cm between
the chest wall and visceral pleural line. He is uncomfortable but not in distress.
Oxygen saturation is 95% on ambient air; heart rate is 95. There is no midline shift
on the chest film; pulsus paradoxus is normal. This is the first time this has
happened to him. He has always been healthy.
You administer 100% humidified oxygen. What's the bestcourse of action over the
next several hours?
A. Continue 100% oxygen and obtain another chest film.
B. Perform thoracentesis, aspirating as much air as possible.
C. Place a Heimlich valve in the anterior chest; discharge home.
D. Admit; place a chest tube to -20 cm wall suction.
E. Admit; consult thoracic surgery for pleurodesis.
We feelthat B and C are the best answers.
This is a primary spontaneous pneumothorax (occurring without a clear cause,
in the absenceof lung disease). Although administering 100% oxygen would result
in slow resorption of the air and lung re-expansion, this is recommended only for
small pneumothoraces. (The size of a pneumothorax is notoriously difficult to
estimate on a chest film, but <3 cm between the lung edge and the chest wall has
been suggested as a cut-off.)
The besttreatment for the patient would be aspiration of air with a standard
thoracentesis kit. After removal of air, leave the catheter in place with a closed
stopcockand monitor for 6 hours. If a repeat chest film shows success and he does
not live in a remote area, he can go home (without the catheter). No follow-up is
necessary, unless symptoms return.
36
Alternatively, a Heimlich (one-way) valve could be attached to the thoracentesis
catheter and he could go home afterward, returning in a few days for repeat film
followed by removal of the catheter and valve.
Chest tube placement could be performed, but would arguably be invasive and
unnecessary, unless thoracentesis is unsuccessful(persistent air aspiration after 4
liters removed, suggesting a bronchopleural communication / leak).
About half of those with PSP will never develop another pneumothorax.
Pleurodesis could be considered for recurrent pneumothoraces
Question 17 of 82 (score?)
You are consulted by the ED for 40-year-old man presenting with shortness of
breath and dry cough. He supervises construction and has been on-site for
excavations in Tennessee, Ohio, Arizona, and Alabama, all in the last 2 months.
He was diagnosed with pneumonia at an urgent care center 2 weeks ago and
treated with levofloxacin for 1 week, with no improvement. Then he was
prescribed prednisone 20 mg daily for 1 week, with steady worsening of his cough
and subjective fevers.
His temperature is 101, pulse 105, respirations 20, blood pressure 125/85. Oxygen
saturation is 94% on room air. White blood cell count is 13,000 with 25%
eosinophils. A chest X-ray shows patchy multifocal pneumonia.
You schedule him for bronchoscopy. What do you most strongly recommend the
ED physician provide now?
A. Methylprednisone 125 mg IV.
B. Itraconazole.
C. Piperacillin and vancomycin.
D. Amphotericin B.
E. Heparin.
We feelthat B is the best answer.
In this patient with a pneumonia that has been unresponsive to broad-spectrum
antibiotics and a courseof corticosteroids, with travel & exposure history
suspicious for fungal infection, empiric antifungal therapy should be provided.
37
Coccidioides is the only dimorphic fungus that causes peripheral eosinophilia.
Itraconazole or fluconazole are the preferred treatments for non-life-threatening
coccidioidomycosis, with amphotericin B reserved for severe or disseminated
infections. These agents are also effective treatment for histoplasmosis and
blastomycosis (with ampho-B reserved for severe infections). There is insufficient
data to supportthe use of voriconazole or posaconazole.
Diagnosis of coccidiodomycosis is strongly suggested by positive serology,
because most people lose seropositivity to coccidioides within 3 months of a
resolved infection. (Positive serologies are also useful in acute histoplasmosis, but
not in blastomycosis.)Negative serology does not rule out any fungal infection,
because serology is insensitive in early infection. A new urine antigen test to
coccidioides is available and should be checked if serologies are negative. Urine
antigen tests are also available for histoplasmosis and blastomycosis (there is
cross-reactivity making these assays only ~80% specific, but it's of no clinical
consequencesince the treatments for both are the same).
Histopathology / biopsycan demonstrate fungal elements using proper staining
(e.g., spherules in coccidioidomycosis), orgranulomas in histoplasmosis. Culture
from BAL or biopsy are high-yield but can take weeks to grow. Apparently,
bronchoalveolar lavage is safe in coccidioidomycosis (although I'd wear an N95
mask and do it in a negative pressure room in suspected cases anyway), but
handling cultures is dangerous and requires special measures; the lab should be
notified if coccidioides is suspected.
Regarding the other answer choices:eosinophilic pneumonia should be considered,
but the lack of responseto an initial courseof steroids (albeit low dose)and the
risk factors for fungal infection would make high dosesteroids ill-advised.
Pulmonary embolism would not likely present with multifocal opacities.
Question 18 of 82 (score?)
Friends of a 21-year-old woman call 911 becauseshe is found unconscious alone
in a room at a party. When EMS arrived, the woman had a GCS scoreof 8, pulse
of 65, and respiratory rate of 12 / min, and was intubated in the field. She was
brought to the emergency department, where you see her an hour later. She is now
lethargic but arousable, moves all extremities and opens her eyes to voice. Pulse is
78, and respirations 18 / min. Physical examination, ECG, and chest film are all
normal.
38
Friends say there was alcohol but no drug use other than marijuana at the party.
They say their friend is a full-time college student who only drinks when at parties.
Ethanol level, chemistry panel and toxicology screen, and other labs are all
pending.
What is the bestnext step?
A. Give flumazenil
B. Give naloxone
C. Place a nasogastric tube and give activated charcoal
D. Observe, with a plan to extubate and discharge from the ED within a few hours
E. Give calcium gluconate
We feelthat D is the best answer.
This young woman is likely intoxicated with ethanol. She seems to be recovering
steadily. In the absence of evidence of other intoxications or poisoning, the fewer
aggressive interventions she undergoes, the better. It could be argued that since she
had a normal respiratory rate and other vital signs, with no evidence of trauma, she
didn't need to be intubated at all. The safety of forgoing intubation in severely
alcohol-intoxicated patients with GCS < 6 was demonstrated in a sample of 405
drunken revelers needing medical attention during Oktoberfestin Munich in 2004.
Other intoxications or poisonings should be considered, but should only be treated
when there is suspicion of a specific ingestion/intoxication. The emergency
department has done that by ordering appropriate labs and tests and by
interviewing her friends.
Naloxone should be given where there is known or suspected opioid toxicity.
Flumazenil should not be administered, even in cases of benzodiazepine overdose,
mainly becauseof its potential for causing withdrawal seizures. Thiamine should
be given in chronic alcohol abusers, but not necessarily to all intoxicated patients.
Activated charcoal is useful in gastric decontamination after ingestion of high-
molecular weight compounds (e.g., acetaminophen or other oral medications), but
should not be administered unless an ingestion is suspected. Charcoal's efficacy
rapidly falls after the first hour after ingestion; evidence for its benefits is low,
overall. Hemodialysis can rapidly remove small molecular weight compounds,
such as methanol and ethylene glycol (and their toxic metabolites) and lithium.
39
(Table: Common Toxidromes Signs & Symptoms, from fmpe.org)
Question 19 of 82 (score?)
Sally K. has COPD with an FEV1 of 48% predicted. You referred her to a lung
rehabilitation program last year, which she completed and which made her feel
better. When her insurer stopped paying for the visits, though, she stopped the
program and returned to a sedentary lifestyle.
She comes to see you today and wants to know what kind of exercise program
would be bestfor her now. She insists on starting only one form of exercise.
What exercise program do you recommend?
A. Aerobic exercise using the legs (e.g., walking)
B. Unsupported arm exercise, low weight, high repetitions
C. Unsupported arm exercise, high weight, low repetitions
D. Inspiratory muscle training
E. Recommend holding off until a cardiac stress test can be obtained
We feelthat A is the best answer.
Aerobic exercise should be the cornerstone of any exercise program for someone
with COPD. Increased cardiopulmonary fitness due to aerobic exercise is believed
to be the major source of benefit from pulmonary rehabilitation, as this patient has
experienced herself.
Aerobic exercise has multiple other beneficial effects on mood, reduced
cardiovascular risk, improved self-image and quality of life. These benefits have
been demonstrated in over 20 randomized controlled trials (although, it should be
noted, benefits were inconsistent & heterogeneous across trials). One study also
shows improvement in muscle mass associated with aerobic exercise in pulmonary
rehab.
The aerobic exercise component of pulmonary rehabilitation should ideally be
lifelong; its short-term nature is a function of the limits set by the payer system. (In
2011, Medicare paid outpatient centers $28 per rehabilitation session, per patient --
which may help explain the low availability of pulmonary rehab programs in many
areas.)
40
Limited studies show that people with COPD are quite inactive. There is no proven
effective method of motivating people to exercise. Some experts recommend tying
exercise to social interaction by exercising with a friend. Varying the routine might
help. Simple brisk walking is all that's needed to get the benefits of exercise, but if
this patient wants to try something "new," there are endless variations on aerobic
exercise, with programs offered through fitness centers, on TV, and online.
Many patients believe that dyspnea is in itself dangerous, and need reassurance that
mild sustained dyspnea during exercise is safe (& in fact is the goal).
Inspiratory muscle training has shown benefit in some trials but should be
considered adjunctive. By no means should it replace aerobic exercise. Strength
training is also beneficial, and can be added to an aerobic exercise program in
highly motivated patients.
Question 20 of 82 (score?)
A 65-year-old man presents with symptoms of myasthenia gravis and a chest CT is
performed, showing a large anterior mediastinal mass. A needle biopsyconfirms
thymoma. The mass is locally advanced, compressing the superior vena cava
(SVC) but not clearly invading the SVC or the pericardium. He has mild COPD
with FEV 70% predicted with no cardiac history.
Which of the following statements is true?
A. An attempt at surgical resection should be made.
B. The mass is unresectable; refer for radiation.
C. Myasthenia symptoms suggest a poorprognosis.
D. Neoadjuvant chemotherapy could improve resection and survival.
E. Absence of malignant features on histology is the strongest predictor of
survival.
You answeredD.
We agree that A and D are the best answers.
Thymomas and thymic carcinomas are neoplasms which may arise from the
thymus. In any case where there is not clear invasion into mediastinal structures
41
(blood vessels, pericardium, pleura), surgical resection should be attempted.
Retrospective series strongly suggest surgical resection offers the bestcure,
although there are no randomized controlled trials of surgery vs. nonsurgical
treatments. This patient's mild lung disease should not preclude surgery.
The histopathology of thymic neoplasms is complex and has an uncertain
contribution to prognosis, because histology is heterogeneous within the tumor
(e.g., carcinoma and thymoma may exist simultaneously in different areas).
Invasion through the thymic capsule into surrounding tissues does diminish
expected survival. However, even with a small degree of local invasion, 5-year
survival with resection is >80%. There are multiple staging systems, with the
Masaoka system most widely used.
Myasthenia gravis (MG) symptoms are present in up to 50% of people with
thymomas, but are rare in thymic carcinoma. Symptoms of MG are associated with
less advanced disease, and resection of the thymoma reduces myasthenia
symptoms (without eliminating them) in most patients.
Palliative radiation is appropriate for cases of unresectable disease. Adjuvant
radiation therapy after resection is strongly recommended after incomplete
resection. After cases of complete resection, adjuvant radiation has an unlikely
benefit except perhaps in stage III disease.
Thymomas are chemotherapy-sensitive. Neoadjuvant chemotherapy may be used
for large bulky masses that on imaging, appear too technically difficult to resect
completely. There are multiple chemotherapy regimens, most of which are based
on cisplatin (e.g., adding doxorubicin, cyclophosphamide and prednisone).
Followup should continue for at least 10 years. Chemotherapy is the preferred
treatment for recurrent or metastatic disease.
Question 21 of 82 (score?)
A 40 year old man underwent bilateral lung transplantation 8 months ago for cystic
fibrosis. He is both a CMV and EBV mismatch (donorseropositive, recipient
42
seronegative). He has done relatively well, but today presents with weight loss,
fever, and night sweats for 2 weeks. A chest CT shows more than 10 lung nodules
bilaterally, varying in size between 0.5 and 4.0 cm, and and mild mediastinal
lymphadenopathy. He is taking tacrolimus, mycophenolate mofetil, and
prednisone.
He undergoes CT-guided biopsy of a large peripheral nodule, and it demonstrates
innumerable clustered lymphocytes. Flow cytometry establishes a polyclonal B-
cell population. Culture of the biopsy is pending. An interferon-gamma release
assay is also pending.
What's the best next step?
A. Initiate anti-bacterial and anti-tuberculosis therapy, pending culture results.
B. Initiate ganciclovir therapy.
C. Reduceimmunosuppression.
D. Perform bronchoscopywith EBUS-guided biopsy of mediastinal lymph nodes.
E. Consult thoracic surgery for mediastinoscopy.
We feelthat C is the best answer.
Post-transplant lymphoproliferative disorder (PTLD) results from EBV infection of
lymphocytes (of donoror recipient origin), inducing them to proliferate and
aggregate where they shouldn't. More than 80% of cases involve B-cell
proliferation; 15% are from T-cells. PTLD occurs in 2-9% of lung transplant
recipients, mostly among those who were EBV-seronegative before transplant.
Multiple well-formed lung nodules is the usual presentation, often with
constitutional symptoms. PTLD tends to present at an advanced stage in lung
transplant recipients. It may involve other organs (including the central nervous
system, abdominal organs, and skin). Pathology demonstrates polyclonal
proliferation of lymphocytes. (Non-Hodghkin lymphoma can also result, which
would result in a monoclonal lymphocyte population on biopsy.)
Reduction in immunosuppression permits the host immune system to suppress
EBV replication and can result in regression of lung lesions and symptoms in
PTLD. However, this increases the risk of acute rejection. Other treatments have
been reported to be successfulin case reports/series:
 Surgical excision
 Radiation therapy
43
 Chemotherapy
 Extracorporeal photochemotherapy
 Retransplantation
 Ganciclovir, rituximab, interferon alfa
Overall, the prognosis of PTLD is guarded, due to the risks of recurrence, and
rejection with reduction in immunosuppression.
Question 22of 82 (score?)
This 48 year old man had right middle and lower lobectomies last year for non-
small cell lung cancer. He did relatively well since, but has now been admitted
from oncology clinic with a fever and a high white blood cell count, and BP of
95/60 with a HR of 125. He is not immune-suppressed and last saw a doctor4
months ago. Doses of levofloxacin and azithromycin have been given.
44
45
46
As the best next step, you recommend:
A. Thoracentesis
B. Bronchoscopywith bronchoalveolar lavage
C. Tube thoracostomy
D. Expedited radiation and salvage chemotherapy
E. Change antibiotics to piperacillin/tazobactam, vancomycin and amikacin.
We feelthat C and A are the best answers.
This may be an empyema, and should be considered one until proven otherwise.
The contrast-enhancing, thickened visceral and parietal pleura on the CT images
are a good example of the "split pleura sign," suggestive of empyema or
47
hemothorax. (A thickened pleura can also be present after lobectomy alone, so this
sign is less specific here.) Thoracentesis would not be inappropriate, but we felt
definitive drainage was best given the patient's sepsis.
Note that the large apparent effusion on scoutchest CT is nearly all due to the
opacity created by an elevated liver/diaphragm due to volume loss after his large
lung resection (see the liver at the level of the heart, on axial cuts). Ultrasound
would help avoid sticking this with a thoracentesis needle. The actual empyema
volume is small. (Cultures of the pleural fluid grew S. pneumoniae). This patient
got a chest tube; these images were actually post-chesttube removal (the original
CT showed an identical-sized fluid collection), which is likely responsible for the
gas bubbles in the effusion.
Without recent health-care contacts or immune suppression, empiric antibiotics for
community-acquired pneumonia are appropriate, and bronchoscopywould not be
expected to help. Even if the effusion were malignant, drainage would be indicated
before further cancer treatments.
Question 22 of 82 (score?)
You've just diagnosed a 32 year old woman with idiopathic pulmonary arterial
hypertension (IPAH) based on the right heart catheterization results below. She
had been lost to followup after her initial evaluation months ago; she now has
dyspnea at rest, lower extremity and sacral edema, and is housebound and
requiring assistance with housekeeping. SaO2 is 85% during ambulation.
Right atrial pressure 14 mm Hg
Right ventricle pressure
69/16 mm
Hg
Pulmonary artery pressure
69/30 mm
Hg
Mean PA pressure 46 mm Hg
48
Pulmonary capillary wedge
pressure
11 mm Hg
Cardiac index
1.6
L/min/m2
Pulmonary vascular
resistance
11 Wood
U
Infusion of nitric oxide reduces the mean PA pressure to 39 mm Hg. The cardiac
index also falls, to 1.3 L/min/m2.
You prescribe warfarin, oxygen therapy, and furosemide. The best choice for
additional therapy is:
A. Oral sildenafil.
B. Oral bosentan.
C. Inhaled treprostinil.
D. I.V. epoprostenol.
We agree that D is the bestanswer.
This patient is WHO Class IV, the most advanced stage of PAH, with dyspnea at
rest and evidence of right heart failure on exam (anasarca). U.S. and European
expert guidelines recommend intravenous prostacyclin analogues (e.g.,
epoprostenol, treprostenil) for WHO Class IV PAH, as there is a larger bodyof
evidence for efficacy with these agents (particularly epoprostenol)than for oral or
inhaled agents.
Vasodilator agents have not been compared well head-to-head, and for less-severe
PAH (WHO class III and below) various agents (oral, IV, or inhaled) are
considered appropriate, depending on the patient's particular severity of disease,
personal preferences, and other risk factors that may be present.
The European Society of Cardiology's 1A recommendations (strong, based on
high-quality evidence) for PAH are:
49
Evidence
level
WHO class II WHO class III
WHO class
IV
I - A
Ambrisentan,
bosentan, sildenafil
Ambrisentan, bosentan, sitaxetan,
sildenafil, epoprostenolIV, iloprost
inhaled
Epoprostenol
IV
Warfarin, oxygen, and diuretic therapy should be standard treatment for advanced
IPAH, barring contraindications. A vasodilator responseis indicated by a drop in
mean PA pressure by 10 mm Hg to less than 40 mm Hg, without a drop in cardiac
output.
Question 23 of 82 (score?)
You are asked your opinion about a 52 year old woman with pleural effusions. She
has decompensated congestive heart failure, but because her effusions were
asymmetric, the admitting physician performed thoracentesis. 1,500 mL of straw-
colored fluid were removed from the left hemithorax, with relief of dyspnea. Since
admission, the patient has received diuretics with further reduction in dyspnea.
Your colleague would like help interpreting the results of the pleural fluid studies:
 Glucose 85 (normal)
 pH 7.38 (normal)
 Protein 4.0 g/dL (serum: 7.4; ratio = 0.56)
 LDH 230 (serum: 290; ratio = 0.79)
 N-terminal brain natriuretic peptide of pleural fluid 1,800 pg/mL
Based on the information you have, what's the best way to describe this effusion?
A. It's exudative, because the findings meet Light's criteria.
B. It's transudative, because the pH and glucose are normal.
C. Tube thoracostomywill likely be necessary for complete drainage.
D. It's due to congestive heart failure, as demonstrated by the clinical picture and
markedly elevated NT-BNP.
E. It's not possible to say, from the information provided.
50
We feelthat D is the best answer.
Light's criteria* are sensitive, but not specific for exudative pleural effusions.
About 25% of transudative effusions may be misclassified as exudates by the
criteria. The transudative effusions of congestive heart failure patients, drained
after diuretic therapy is given, are commonly misclassified as exudates this way
Presumably, the diuretics remove water and increase the concentration of protein
and LDH in pleural fluid.
A gradient of more than 3.1 g/dL in protein concentration between the pleural fluid
and serum (as seen here) correctly reclassifies many effusions as transudates. In
effusions due to CHF, N-terminal BNP in the pleural fluid > 1,500 pg/mL is
better than the serum-pleural protein gradient at accurately reclassifying "exudates"
as transudates. Notably, pleural fluid BNP is much less reliable than pleural fluid
NT-BNP for this purpose.
Properly reclassifying a CHF-related effusion as a transudate spares the patient
unnecessary further testing and treatment. Therapeutic thoracentesis could be
provided to relieve severe dyspnea, but tube thoracostomygenerally should not be
performed.
* Light'scriteria: Pleural fluid-to-serum protein ratio > 0.5; LDH in pleuralfluid
> 60% of serum value; LDH > 2/3 the upper limit of normalfor serum.
Question 24 of 82 (score?)
A victim of a house fire is brought to your ICU from the ED. He was found in his
garage where he has a sophisticated workshop, according to a neighbor, but no one
knows what chemicals may have been present. He was comatoseand was intubated
in the field. He is only minimally burned; a rapid bronchoscopyin the ED showed
no serious mucosal injury.
His vital signs: HR 110, BP 100/70, easily bag-mask ventilated, oxygen saturation
of 94% with an FiO2 of 0.90.
His ABG is 7.20 / 44 / 80, with 24% methemoglobin. Lactic acid is 7.5. The ED
MD placed a central venous catheter, and central venous oxygen saturation is 90%,
with a pO2 in venous blood of 68.
51
What therapy do you prioritize first?
A. Hydroxycobalamin and sodium thiosulfate
B. Amyl nitrite
C. Pralidoxime
D. Hyperbaric oxygen
E. Benzodiazepines
You answeredA.
We agree that A is the bestanswer.
Up to 35% of fire victims have cyanide poisoning. Incinerated plastics, rubber,
polyurethane, and melamine (in household goods)can all release lethal inhaled
doses ofcyanide. Sodium nitroprusside in prolonged infusions or in people with
renal failure is another cause. Cyanide disrupts mitochondrial oxidative
phosphorylation and aerobic metabolism of glucose. Signs of cyanide poisoning
include a high venous pvO2 with a low venous-arterialpO2 gradient (due to
inadequate peripheral tissue oxygen use) and an anion-gap lactic acidosis.
Symptoms can affect virtually all organ systems, the most serious including coma;
pulmonary edema with respiratory failure; and hypertension, followed by
hypotension, bradycardia, ventricular dysrrhythmias and cardiopulmonary
collapse.
Cyanide poisoning is rare but lethal, and definitive diagnosis (by serum cyanide
level) may require hours. Therefore, empiric therapy should be given rapidly if
cyanide toxicity is suspected. Sodiumthiosulfate is a sulfur donor that transforms
cyanide to a less-toxic compound, and should be given to all suspected victims.
Amyl nitrite is a standard co-antidote, working by producing methemoglobin,
which then binds and detoxifies cyanide. (Sodium nitrite or dimethylaminophenol
also induce methemoglobinemia and are alternate agents; methylene blue releases
free cyanide and should not be given if cyanide toxicity is suspected). This patient
already has serious methemoglobinemia likely due to carbonmonoxide inhalation,
however, and worsening this could be catastrophic. Hydroxycobalamin directly
binds and detoxifies cyanide, without producing methemoglobin, and should be
administered with sodium thiosulfate. Hyperbaric oxygen may reduce cognitive
sequelae of carbon monoxide poisoning (although a Cochrane review casts doubt
on this), but shouldn't take first priority.
52
Pulse oximetry and arterial blood gas can be unreliable in cases of severe smoke
inhalation. Carbon monoxide poisoning increases carboxyhemoglobin, falsely
raising SaO2; methemoglobinemia tends to push SaO2 toward 85% (due to equal
absorption of oxyhemoglobin and deoxyhemoglobin wavelengths).
Methemoglobinemia can also cause ABG to overestimate SaO2, but is helpful in
showing the levels of carboxyhemoglobin and methemoglobin. Co-oximetry
measures 4 wavelengths of light (compared to regular oximetry's 2) and is
necessary to accurately identfy the oxyhemoglobin level (saturation) in cases of
carbonmonoxide poisoning.
Cyanide affects virtually all bodytissues, attaching itself to ubiquitous
metalloenzymes and rendering them inactive. Its principal toxicity results from
inactivation of cytochrome oxidase (at cytochrome a3), thus uncoupling
mitochondrial oxidative phosphorylation and inhibiting cellular respiration, even in
the presence of adequate oxygen stores. Cellular metabolism shifts from aerobic to
anaerobic, with the consequent production of lactic acid. Consequently, the tissues
with the highest oxygen requirements (brain and heart) are the most profoundly
affected by acute cyanide poisoning.
Smoke inhalation, suicidal ingestion, and industrial exposures are the most
frequent sources of cyanide poisoning.
Smoke inhalation
Studies in France, Sweden, and Scotland, as well as in the United States, have
documented smoke inhalation as an important sourceof cyanide poisoning.
Individuals with smoke inhalation from enclosed spacefires who have sootin the
mouth or nose, altered mental status, or hypotension may have significant cyanide
poisoning (blood cyanide concentrations >40 mmol/L or approximately 1 mg/L).
Many compounds containing nitrogen and carbon may producehydrogen cyanide
(HCN) gas when burned. Some natural compounds (eg, wool, silk) produceHCN
as a combustion product.
Household plastics (eg, melamine in dishware, acrylonitrile in plastic cups),
polyurethane foam in furniture cushions, and many other synthetic compounds
53
may producelethal concentrations of cyanide when burned under appropriate
conditions of oxygen concentration and temperature.
Intentional poisoning
Cyanide ingestion is an uncommon, but efficacious, means of suicide, often
involving cyanide salts found in hospital and research laboratories. Not
surprisingly, individuals in certain occupations, suchas health-care and laboratory
workers, are at risk for suicidal ingestion of cyanides.
Industrial exposure
Countless industrial sources ofcyanides exist. Cyanides serve an extremely
important role in the metal plating and recovery industries. In addition, industry
uses cyanides in the manufacture of plastics, as reactive intermediates in chemical
synthesis, and as solvents (in the form of nitriles).
Exposure to salts and cyanogens occasionally causes poisonings; however, a
significant risk for multiple casualties occurs when these products comeinto
contact with mineral acids because HCN gas is produced. Water contactwith the
soluble salts (eg, potassium, sodium cyanide) also may liberate HCN.
Iatrogenic exposure
Sodium nitroprusside, when used in high doses orover a period of days, can
producetoxic blood concentrations of cyanide. Patients with chronic renal failure,
pediatric patients, and those with low thiosulfate reserves (eg, malnourished,
postoperative) are at increased risk for developing symptoms, even with
therapeutic dosing. Resultant confusion and combativeness initially may be
mistaken as intensive care unit (ICU) syndrome (ie, sundowning). Problems may
be avoided by coadministration of hydroxocobalamin or sodium thiosulfate.
Ingestionof cyanide-containing supplements
Ingestion of cyanide-containing supplements is rare. Amygdalin (synthetic laetrile,
also marketed as vitamin B-17) contains cyanide and can be found in the pits of
many fruits, such as apricots and papayas; in raw nuts; and in other plants (lima
beans, clover, and sorghum).
The substancewas thought to have anticancer properties due to the action of
cyanide on cancer cells. However, laetrile showed no anticancer activity in human
54
clinical trials in the 1980s and is not available in the United States,[2] although it
can be purchased on the Internet.
EmergencyDepartment Care
Initial emergency department care for patients with cyanide exposure is identical to
that provided in the prehospital phase.
Provide supportive care, including the following:
 Airway control, ventilation, 100% oxygen delivery
 Crystalloids and vasopressors, as needed, for hypotension
 Sodium bicarbonate titrated according to arterial blood gas (ABG) and
serum bicarbonate level
Decontaminate the patient with removal of clothing/skin flushing and/or activated
charcoal (1g/kg), as appropriate. Activated charcoal should be given after oral
exposure in alert patients who are able to protect the airway or after endotracheal
intubation in unconscious patients. Remember to protectthe health-care provider
from potential contamination.
Administer cyanide antidotes if the diagnosis is strongly suspected, without
waiting for laboratory confirmation. The antidotes include hydroxocobalamin
(Cyanokit) and the Cyanide Antidote Kit, which includes amyl nitrite pearls,
sodium nitrite, and sodium thiosulfate.
Cyanokit
Hydroxocobalamin (Cyanokit), routinely used in Europe, has been approved by the
US Food and Drug Administration (FDA) for treating known or suspected cyanide
poisoning.[7, 8]
Hydroxocobalamin combines with cyanide to form cyanocobalamin (vitamin B-
12), which is renally cleared. Hydroxocobalamin administration resulted in faster
55
improvement in mean arterial pressure but similar mortality and serum acidosis, as
compared with sodium nitrite, in animals.[9]
A repeat doseof hydroxocobalamin and/or coadministration of sodium thiosulfate
(through a separate line or sequentially) have been suggested to improve
detoxification and are recommended in patients with continuing elevated lactate
levels or continuing signs of cyanide toxicity.[10, 11]
Adverse effects of hydroxocobalamin administration include transient hypertension
(a benefit in hypotensive patients), reddish brown skin, mucous membrane and
urine discoloration, and rare anaphylaxis and anaphylactoid reactions. It also
interferes with co-oximetry (about a 5% increase in carboxyhemoglobin levels)
and blood chemistry testing (bilirubin, creatinine kinase and possibly liver
enzymes, creatinine, phosphorus,glucose, magnesium, and iron levels) due to its
bright red color.[12] It can also interfere with hemodialysis.[13]
Certain medications should not be administered simultaneously or through the
same line as hydroxocobalamin, including diazepam, dopamine, dobutamine, and
sodium thiosulfate.
Cyanide Antidote Kit
The Cyanide Antidote Kit contains amyl nitrite pearls, sodium nitrite, and sodium
thiosulfate. Amyl and sodium nitrites induce 15-20% methemoglobinemia in red
blood cells, with methemoglobin combining with cyanide and releasing
cytochrome oxidase enzyme. Inhaling crushed amyl nitrite pearls is a temporizing
measure before IV administration of sodium nitrite.
Sodium thiosulfate enhances the conversion of cyanide to thiocyanate, which is
renally excreted. Thiosulfate has a somewhat delayed effect and thus is typically
used with sodium nitrite for faster antidote action.
Avoid the nitrite portion of the kit in patients with smoke inhalation unless
carboxyhemoglobin concentration is very low (< 10%). The induction of
methemoglobinemia from the nitrites, in addition to present
carboxyhemoglobinemia, significantly reduces the oxygen-carrying capacity of
blood.
In patients with preexisting anemia, the sodium nitrite doseneeds to be reduced
dosein proportion to the hemoglobin concentration. Consult a regional toxicology
center for appropriate dosing.
56
Vasodilatation leading to hypotension is another adverse effect of the Cyanide
Antidote Kit.
Question 25:
A 51 year old woman is referred to you by her infectious disease physician for a
second opinion. She has had cough and worsening dyspnea for a few months. For
this, she underwent open lung biopsy elsewhere; transbronchial biopsies showed
noncaseating granulomas. She was diagnosed with sarcoidosis, and her ID doctor
wants your OK before starting steroids.
She has HIV, well-controlled on antiretroviral therapy with a CD4 count of 550.
Exam is notable for oxygen saturation of 92% on ambient air and faint crackles at
the lung bases. PFTs showa mild restrictive defect and moderate diffusion
impairment. Chest X-ray is within normal limits and shows no hilar
lymphadenopathy. Chest CT images are below.
57
58
You review the pathology results, which describe"noncaseating granulomas and
monotonous sheets of T and B lymphocytes in the lung interstitium."
What's the most likely diagnosis?
A. Lymphocytic interstitial pneumonia.
B. Primary pulmonary lymphoma.
C. Secondarypulmonary lymphoma.
D. Sarcoidosis.
E. Pneumocystis pneumonia.
We feelthat A is the best answer.
Lymphocytic interstitial pneumonia is an uncommon condition that usually occurs
in association with some form of immune dysfunction or dysproteinemia, such as
autoimmune disease, HIV or EBV infection, or multiple myeloma. A few cases
may occurwithout such comorbidities. In LIP, sheets of polyclonal T and B
lymphocytes infiltrate the lung interstitium, causing a variable degree of
dysfunction.
Clinical:Cough and dyspnea are most common, but systemic/constitutional
symptoms can occur. Hypoxemia and clubbing are common in more severe
disease.
59
Diagnosis:Requires open lung biopsyfor confirmation. Besides sheets of
polyclonal lymphocytes, plasma cells and macrophages, giant cells, noncaseating
granulomas, amyloid deposition, lymphoid germinal centers, and fibrosis may all
be found on biopsy. Dominant findings are in the interstitium but may "spill" into
the airspaces.
Imaging:Chest film with reticular opacities later coalescing into
airspace/interstitial pattern. Chest CT with centrilobular nodules; ground glass;
thickened interlobular septa; later with cystic airspace disease and consolidation.
Late honeycombing is possible.
PFTs:Usually restrictive, with a diffusion impairment.
Bronchoscopy: Lavage shows an abundant T-cell lymphocytosis.
Treatment:Corticosteroids have been reported to be beneficial in case series.
Prognosis: Unpredictable, varying from indolent to progressive and fatal. Some
patients' pulmonary disease may progress despite corticosteroids. Since LIP is
usually a feature of an immune condition, prognosis is believed to be dependent on
the underlying condition. Although LIP may progress to a low-grade pulmonary or
systemic lymphoma, this seems to be uncommon (~5%).
Sarcoidosis is unlikely in the absence of lymphadenopathy, and does not have
sheets of lymphocytic infiltration on biopsy. Lymphomas are generally defined as
monoclonal, not polyclonal B or T cell populations. Pneumocystis pneumonia is
unlikely in well controlled HIV at this CD4 count.
60
Question 26of 82 (score?)
A patient taking pramipexole for restless legs syndrome for the past 2 years
complains of symptoms worsening and occurring earlier in the day, and feeling
symptoms in her arms as well as legs. Ferritin is 125 (normal).
What do you recommend?
A. Add another doseof pramipexole earlier in the day.
B. Switch to a benzodiazepine 1-2 hours before bed.
C. Switch to levodopa 1-2 hours before bed.
D. Start iron replacement therapy.
You answeredA.
We agree that A and B are the best answers.
Augmentation of restless legs syndrome (increase in symptom severity,
occurrence earlier in the day, spreading to other bodyparts) is less common with
dopamine agonists than with levodopa, but can occurand is more common after
years of therapy.
Interestingly, increasing dopamine-agonist therapy by adding a doseearlier in the
day can diminish augmentation symptoms, and would be reasonable to try.
Switching to another dopamine-agonist (e.g. ropinirole) would be reasonable.
Levodopais a viable option but is recommended by an expert advisory panel to be
used only for intermittent RLS.
Benzodiazepines at night (diazepam, clonazepam) are a well-established therapy
for RLS and could be substituted for the pramipexole.
Carbergoline works well for RLS but is used rarely (it causes cardiac valve disease
when used at high doses for Parkinson's).
Gabapentin, opioids (codeine, methadone, tramadol, etc), pregabalin, and other
drugs have been shown beneficial for RLS in studies of varying quality.
61
Iron deficiency should be treated if present in people with RLS, but a normal
ferritin rules it out in the absence of acute illness. Iron deficiency itself is not
known to cause augmentation.
Question 27 of 82 (score?)
It's 1962, and you work for an insurance company. It's a boring job but they assign
you an interesting project. They're pretty sure smoking causes lung cancer, and
they want to prove it so they can defend raising premiums on all their smokers.
What's the most appropriate study design to answer the question of whether
smoking causes lung cancer?
A. Randomized controlled trial.
B. Case series review.
C. Cohortstudy.
D. Case-controlstudy.
E. Telephone survey.
We feelthat C is the best answer.
Randomized controlled trials have the greatest strength in proving causation.
However, randomizing people to smoke or not to smoke would be unethical.
A cohort study is the next best study design to prove causality. In a prospective
cohortstudy, one cohortof subjects with the exposure of interest (i.e., smokers)
and one without (non-smokers) are followed over time for the development of the
outcome of interest (lung cancer). With large enough cohortsizes, other potentially
confounding factors can be controlled for, and the independent contribution of the
exposure determined with a high degree of confidence. The main downside of
cohortstudies is that they take a long time (years).
In a case-controlstudy, cases (i.e., people with lung cancer) are identified within a
database, and are compared against controls (people without lung cancer) across
multiple factors. Statistical analysis identifies factors (e.g., smoking, age, diet) that
are distributed differently between cases and controls. A factor that is much more
prevalent in lung cancer cases (such as smoking) is then identified as a risk factor.
Case-controlstudies can be donerapidly and cheaply compared to randomized
62
trials and cohortstudies, but can only establish associations between exposures and
outcomes;case-controlstudies can't prove causation.
Case series and telephone surveys aren't controlled, and can't establish causation at
all. At most, they might generate a hypothesis for a well-conducted clinical study
of one of the above types.
Question 29 of 82 (score?)
A 58 year old man in your clinic has chest pain and palpitations. He becomes
diaphoretic; blood pressure is 80/60.
According to ACLS algorithms, what is the next thing you should do?
A. Perform synchronized electrical cardioversion.
B. Give a chewed 160-325 mg aspirin.
C. Obtain IV access.
D. Give amiodarone 300 mg IV.
E. Perform unsynchronized cardioversion (defibrillation dose).
You answeredA.
We agree that A is the bestanswer.
This is symptomatic or unstable ventricular tachycardia, based on hypotension
(systolic BP < 90) and symptoms (chest pain). Current ACLS guidelines (good
through 2015) advise immediate electrical cardioversion. Shocks of 50-100 joules
are appropriate initially, escalating to 200 J if unsuccessful.
63
Question 30of 82 (score?)
You are consulted for pleural effusions in a 61 year old man directly admitted to
the hospital an hour ago. He presented with progressive dyspnea and leg swelling
and no other symptoms. He has jugular venous distension, dullness at the lung
bases, and pitting leg edema. Vitals are all normal except for BP 145/90. He has
NYHA III symptoms (dyspneic walking across the room). He has received no
treatments yet.
Chest X-ray shows moderate-sized bilateral pleural effusions and an enlarged
cardiac silhouette. An echocardiogram and labs are pending.
What's the best initial management for this patient's pleural effusions?
A. Diagnostic thoracentesis (unilateral).
B. Diagnostic thoracentesis (bilateral).
C. Therapeutic thoracentesis (bilateral, with a chest film in between).
D. Diuretics and afterload reduction.
E. All of the above.
We feelthat D is the best answer.
Clinically, this patient has congestive heart failure. It's overwhelmingly likely that
the effusions are due to his CHF, and a diagnostic thoracentesis is not necessary. A
few proposedindications for performing diagnostic thoracentesis in people with
CHF-related pleural effusions are:
 Fever (to diagnose a possible pleural infection)
 Grossly asymmetrical effusions
 Chest pain (suggesting an inflammatory process affecting the pleura)
None of these are present here. Therapeutic thoracentesis is reasonable in people
with severe dyspnea due to decompensated CHF, but not as initial therapy before
first-line, noninvasive, medical therapy has been provided (diuretics and afterload
reduction with vasodilator therapy). These may rapidly eliminate the effusions and
symptoms.
64
Question 32 of 82 (score?)
Mr. R, your 76-year old man in bed 3, just experienced a cardiac arrest on ICU day
6, after admission for septic shocklast week. His lack of other medical problems,
along with a reduced need for his norepinephrine infusion and mechanical
ventilatory support, had led you to hope he was improving. This morning, though,
his blood pressure abruptly plummeted and he had a PEA arrest, with recovery of
spontaneous circulation after 9 minutes of ACLS. After the code, you collect your
thoughts and head out of the room to talk to his wife.
After hearing you explain the situation, she asks, "Doctor, what are my husband's
chances of coming home from the hospital to live a normal life with me?" You
gently inform her it will require a few more days to provide the most accurate
prognosis.
At this point in time (immediately post-resuscitation in the ICU), based on large
observational series, what are Mr R's chances for being discharged home with a
good neurologic outcome?
A. 48%
B. 32%
C. 16%
D. 4%
E. It's impossible to say.
You answeredD.
We agree that D is the bestanswer.
Prognosis after a first cardiac arrest occurring in the ICU is poor. An observational
study of almost 50,000 such arrests showed an overall survival to hospital
discharge of 16%. Requiring vasopressors priorto the arrest was a major
discriminator in outcome:
65
 Only 10% of patients on pressors prior to arrest survived to discharge, and
only 4% were discharged home (the others went to rehab or long-term acute
care).
 Among those with PEA/asystole despite pressors,only 1.7% were able to
perform their own activities of daily living at the time of discharge.
 People with ventricular fibrillation or tachycardia not requiring pressors
prior to arrest did much better: 40% survived, 20% went home, and 17%
had good neuro outcomes.
 These data did not provide longitudinal outcomes data post-hospital
discharge (e.g., neurological & functional recovery after 6 months);
however, those outcomes are usually also poor(see below).
Physicians do not usually convey prognosis after cardiac arrest (or critical illness
in general) accurately or effectively to patients' families (according to those
families' responseto surveys). Although such discussions are difficult for all
parties involved, families require and deserve this information as they struggle
with their roles as surrogate decision makers. Although some people discharged to
rehabs or LTACs may recover function, it would be dishonest to give false hope by
suggesting this as anything other than a long shot, especially after an in-ICU
cardiac arrest. Only a minority of people survive one year after LTAC admission,
and many fewer recover to live without needing major assistance.
For example, in a 2009 study of 126 patients receiving tracheostomies for
prolonged mechanical ventilation for various causes of respiratory failure, 93% of
surrogates expected their loved one to survive one year, recovering full function &
independence (71%) and good quality of life (83%), while only 6% of physicians
expected independence and only 4% a good quality of life. But only 26% of
surrogates recalled the doctorproviding any information about survival, quality of
life, and need for caregiving. A third of the families reported it was the doctor's
decision, not theirs, to perform tracheostomy. (The real outcomes at 1 year: 56% of
those patients did survive; 9% regained independence; surrogates reported good
quality of life in 33% of the survivors, or 19% of the original sample. The majority
of survivors were cared for at home, and surrogates reported significant personal
emotional and financial strain.
66
Question 33 of 82 (score?)
You are using airway pressure release ventilation (APRV or "bi-level") for Ms. P,
a 76-year-old woman with ARDS from sepsis. She has been on mechanical
ventilation for 5 days. She is in shockand is on vasopressors. Her ventilator
settings are:
P[high] = 30; P[low] = 0
T[high] = 5 sec; T[low] = 0.6 sec
PEEP = 0; FiO2 = 0.55
Her most recent arterial blood gases are:
pH 7.25, paCO252; paO2 62 (8 hours ago)
pH 7.19, paCO262; paO2 60 (30 minutes ago)
Her ideal bodyweight is 60 kg and she is receiving tidal volumes (Vt) of 360 mL.
Mean airway pressure is 24 mmHg.
The bestthing to do next would be to:
A. Increase FiO2 to 0.65
B. Increase PEEP to 5
C. Decrease T[high] to 4 sec
D. Decrease P[high] to achieve Vt of 5mL / kg
E. Decrease T[low] to 0.4 sec
We feelthat C is the best answer.
Ms P is hypoventilating due to an inadequate minute ventilation being delivered.
Airway pressure release ventilation (APRV or "bi-level") cycles between two
alternating levels of CPAP, P[high] and P[low], whose durations are determined by
T[high] (usually several seconds)and T[low] (usually a fraction of a second). This
pattern maintains a stable high mean airway pressure, keeping alveoli open and
67
augmenting oxygenation--an "openlung" strategy. APRV can also be viewed as a
form of pressure-controlled, inverse-ratio ventilation. Ventilation (exhaled tidal
volume and CO2) occurs during the pressure drop from P[high] to P[low] (hence
"pressure release ventilation"). PEEP is usually set to 0, but the short time at
P[low] creates beneficial auto-PEEP and prevents alveolar collapse. Patients may
take small additional breaths throughout the entire cycle; these Vt improve alveolar
recruitment and oxygenation but contribute little to minute ventilation.
The most effective way to increase minute ventilation in APRV is to decrease
T[high], which increases the frequency of breaths. Although Ms P's ABG is
acceptable for a patient with ARDS, her acidemia is worsening. Many would argue
that hypercarbic respiratory acidemia (down to pH of 7.15 or so)is well-tolerated
and poses little risk. However, the trend is concerning and potentially dangerous
should it continue. This is an easy ventilator change to make with no apparent risk.
Reducing minute ventilation further by reducing delivered tidal volumes would be
unwise when peak and mean pressures are acceptable, and while her respiratory
acidemia is worsening. Increasing PEEP would also reduce her delivered minute
ventilation, because there would be a smaller pressure gradient between P[high]
and P[low]. Decreasing T[low] by 0.2 sec would probably have little effect. Her
oxygenation is fine; changing FiO2 is not necessary.
Question34 of 82 (score?)
A 64 year old man complains of shortness of breath after a successfulcoronary
artery bypass graft surgery 5 weeks prior. He is found to have a large left-sided
pleural effusion (about 50% of the hemithorax) and readmitted to his surgeon's
team. A thoracentesis drains 1,800 mL of fluid, relieving his dyspnea, A follow-up
chest film shows minimal remaining fluid. He had no other presenting symptoms
besides dyspnea, and is asymptomatic now. The fluid was exudative (LDH 400)
and had 1,100 white cells/mL, 71% lymphocytes. He looks and feels better and
wants to go home. The cardiothoracic surgeon asks you your opinion.
What's the best advice to give?
68
A. Stay in the hospital to monitor for reaccumulation and for culture results.
B. Dischargehome; if the fluid reaccumulates we'll justtap it again.
C. Dischargehome; if the fluid reaccumulates we'll place a chest tube.
D. Dischargehome; if the fluid reaccumulates we'll place a pleural drain.
E. Place a chesttube now, as the likelihood of rapid recurrence is high.
You answeredB.
We agree that B is the best answer.
Pleural effusions are common after cardiac surgery. Most are due to inflammation
and anatomic disruption from the surgery itself, and are generally benign. These
are called nonspecific pleural effusions, and may occurearly (<30 days) or late
(>30 days) after surgery. If small, asymptomatic, and developing early after
surgery, nonspecific pleural effusions require no further evaluation.
Late, large (>25% of the hemithorax) and/or symptomatic nonspecific pleural
effusions should be drained and the fluid analyzed in the usual fashion, as
hemothorax, parapneumonic effusions, chylothorax, and other etiologies are also
possible.
The fluid in both early and late nonspecific pleural effusions is usually exudative
and leukocyte-rich, reflecting the causative inflammatory process. Early effusions
are often bloodyand rich in eosinophils and neutrophils; late effusions are more
clear (but exudative) and rich in lymphocytes.
Treatment of nonspecific pleural effusions (if needed) post-cardiac surgery is
drainage with thoracentesis. More aggressive measures are rarely required,
although repeat thoracenteses (up to 3) may be necessary. NSAIDs and
glucocorticoids have also been used in a non-evidence based fashion.
Postpericardiotomysyndrome is a distinct, persistent inflammatory state that
occurs days to weeks after cardiac surgery, and frequently results in pleural
effusions. In contradistinction to nonspecific pleural effusions, pericardial
effusions, fever, chest pain, leukocytosis, pleural and/or pericardial rub are usually
present (similar to / forme fruste of Dressler's syndrome). The pleural effusion may
be small, but is often bloody, exudative, and leukocyte-rich. NSAIDs and/or
glucocorticoids are the mainstay of treatment.
69
Post-CABG effusions that recur and persist after multiple thoracenteses, or
persistent effusions discovered late (after months have passed)may require
decortication to prevent trapped lung and loss of lung function.
Question35 of 82 (score?)
A 51-year-old woman comes to your clinic for follow-up. She has coughed up
colored sputum a few days a week for the past 15 years, very infrequently streaked
with blood (none lately). It may be getting a little worse, slowly. She feels at her
baseline today.
A chest X-ray and high-resolution CT scan show extensive bronchiectasis in the
lower lobes bilaterally. She has diabetes, a history of repeated respiratory illnesses,
and a history of non-Hodgkin lymphoma in remission.She was previously followed
by a pulmonologist and says she was told she does not have asthma. She has three
biological children. She never smoked. Other test results include:
 HIV antibody and TB skin test are negative.
 Total IgElevels are normal.
 Sweat chloride test is normal.
 White cell count is 11 with no eosinophils.
She had a bronchoscopylast year during an exacerbation, showing purulent
secretions. The BAL grew >100K H.influenzaeand she was treated with
antibiotics with a return to her baseline.
What would be the best next test to order to determine the underlying cause of her
bronchiectasis?
A. CFTR mutation testing
B. Quantitative immunoglobulins
C. IgG subclass levels
D. Precipitating serumantibodies to Aspergillus
E. Biopsy of nasalepithelium
We feelthat A and B are the best answers.
70
Because some causes of bronchiectasis are treatable, finding the etiology (when
possible) is important. (Even after extensive workup, as many as 50% of cases of
bronchiectasis have no identifiable cause, though.)
This woman has features suggestive of chronic variable immune deficiency
(CVID), in which impaired B-cell differentiation leads to low immunoglobulin
production. Levels of IgG, and usually IgA and/or IgM are all below the lower
limit of normal. An inadequate specific IgG responseafter intentional vaccination
(e.g., with tetanus & diphtheria booster)can help make the diagnosis.
Most CVID patients have a history of bacterial pneumonia and/or rhinosinusitis
(esp. Pneumococcus, Mycoplasma, and Hemophilus); chronic lung disease and
bronchiectasis are common but their exact prevalences are unknown.
Gastrointestinal disease, autoimmune disease (especially hematologic, e.g. immune
thrombocytopenic purpura), and malignancy (especially non-Hodgkin lymphoma
and gastric cancers), are also common.
Noncaseating granulomas in the liver, lungs, and lymph tissue can occurin CVID,
mimicking sarcoid. CVID is in part a diagnosis of exclusion, and other immune
deficient states must be ruled out.
Certain heterozygous CFTR mutations can cause diffuse bronchiectasis in people
with negative sweat chloride testing and without cystic fibrosis.
Measurement of IgG subclasses should only be performed after first checking total
immunoglobulin levels. Information provided in the question stem makes most of
the other major known causes of bronchiectasis less likely:
Condition Features Diagnosis
Primary ciliary
dyskinesia
Situs inversus in 50%; Infertility
in most men & >50% of women
Exhaled nasalnitric oxide;
Nasal brush biopsy
Cystic fibrosis
7% are diagnosed after age 18;
Pseudomonas, non-TB
mycobacteria; Often upper lobe
bronchiectasis
Sweat chloride test >30;
mutation screening of CFTR
gene
Allergic Asthma; usually (+)blood Elevated total IgEand
71
bronchopulmonary
aspergillosis
eosinophils (+)IgE& IgG to Aspergillus;
central bronchiectasis; skin
test reactive to Aspergillus
Common variable
immunodeficiency
Pneumococcus, Hemophilus,
Mycoplasma infections; see
above
Low levels of IgG and
either IgA or IgM; impaired
responseto vaccines
Airway obstruction
Impaired consciousness
(seizures, strokes, EtOH)
CT scan; bronchoscopy
Immunedeficiency Organ transplant, HIV History
Young's syndrome Sinusitis, azoospermia Exclusion
Alpha-1 antitrypsin
deficiency
Less common alternative
presentation fromemphysema
A1ATlevel
Non-TB
mycobacteria
Can be primary process (esp. in
nonsmoking women age 50+) or
secondary infection in existing
bronchiectasis
Sputumculture x 2, or
bronchoscopic samplex 1,
with radiologic/clinical
compatibility
Question41 of 82 (score?)
A 70 year old man had a right upper lobectomy for lung adenocarcinoma 6 months
ago. He came today to his oncologist for a routine surveillance CT scan, which is
72
shown below. The air-fluid level was somewhat lower on his previous scan 3
months ago. No abnormal fistulous tracts are seen on the remainder of the CT. His
white blood cell count is 9 today. He has "good days and bad" and today is one of
his average days. He went for his usual half-mile walk this morning and drove
himself in. His vital signs are normal. His chronic cough productive of white
sputum is unchanged. The oncologist, your colleague in a multispecialty practice,
asks you what you think.
The bestnext step is:
73
A. Observation
B. 14 day courseof Augmentin; call if fever develops
C. Intravenous broad-spectrumantibiotics
D. Needle aspiration of fluid
E. Bronchoscopy with lavage
We feelthat A is the best answer.
The surgically created cavity left behind after a lobectomy or pneumonectomy will
normally slowly fill up with sterile serous fluid, eventually forming an opacity on
imaging with mediastinal shift toward the surgical side. This is likely part of that
process, although it usually happens in weeks, not months. The late appearance of
fluid justifies consideration that a new infection is responsible. However, in the
absence of signs or symptoms, the images shouldn't by themselves dictate workup
or treatment.
A bronchopleural fistula could be considered (the delayed or chronic form; most
BPFs occurin the early postop period), but would always be associated with
symptoms (fever and cough, nonspecific malaise). Acute/subacute BPFs usually
cause a new relative mediastinal shift to the contralateral side; delayed BPFs may
not, as fibrosis (anchoring things in place) has usually occurred. CT scanning is not
sufficiently sensitive for detecting BPF, and absenceof a visible tract on CT does
not rule out the condition. This man went home after his visit and did fine.
Fleischner Society Recommendations
Incidental Pulmonary Nodule Follow-up
74
Question43 of 82 (score?)
A 46 year old man comes to your clinic for management of his asthma. He takes
high-dose inhaled corticosteroids and a long-acting beta agonist, along with a
leukotriene inhibitor. His adherence and technique are perfect.
He still has symptoms of cough, wheezing, and chest tightness that bother him
most days and nights each week. He is using albuterol daily. The symptoms persist
when he goes on vacation out of state.
Sputum culture is negative. IgE level is 3,600 ng/mL. His primary doctorobtained
imaging and a chest CT, which are shown.
75
76
77
Real-World Boards Cases ,PULMCCM
Real-World Boards Cases ,PULMCCM
Real-World Boards Cases ,PULMCCM
Real-World Boards Cases ,PULMCCM
Real-World Boards Cases ,PULMCCM
Real-World Boards Cases ,PULMCCM
Real-World Boards Cases ,PULMCCM
Real-World Boards Cases ,PULMCCM
Real-World Boards Cases ,PULMCCM
Real-World Boards Cases ,PULMCCM
Real-World Boards Cases ,PULMCCM
Real-World Boards Cases ,PULMCCM
Real-World Boards Cases ,PULMCCM
Real-World Boards Cases ,PULMCCM
Real-World Boards Cases ,PULMCCM
Real-World Boards Cases ,PULMCCM
Real-World Boards Cases ,PULMCCM
Real-World Boards Cases ,PULMCCM
Real-World Boards Cases ,PULMCCM
Real-World Boards Cases ,PULMCCM
Real-World Boards Cases ,PULMCCM
Real-World Boards Cases ,PULMCCM
Real-World Boards Cases ,PULMCCM
Real-World Boards Cases ,PULMCCM
Real-World Boards Cases ,PULMCCM
Real-World Boards Cases ,PULMCCM
Real-World Boards Cases ,PULMCCM
Real-World Boards Cases ,PULMCCM
Real-World Boards Cases ,PULMCCM
Real-World Boards Cases ,PULMCCM
Real-World Boards Cases ,PULMCCM
Real-World Boards Cases ,PULMCCM
Real-World Boards Cases ,PULMCCM
Real-World Boards Cases ,PULMCCM
Real-World Boards Cases ,PULMCCM
Real-World Boards Cases ,PULMCCM
Real-World Boards Cases ,PULMCCM
Real-World Boards Cases ,PULMCCM
Real-World Boards Cases ,PULMCCM
Real-World Boards Cases ,PULMCCM
Real-World Boards Cases ,PULMCCM
Real-World Boards Cases ,PULMCCM
Real-World Boards Cases ,PULMCCM
Real-World Boards Cases ,PULMCCM
Real-World Boards Cases ,PULMCCM
Real-World Boards Cases ,PULMCCM
Real-World Boards Cases ,PULMCCM
Real-World Boards Cases ,PULMCCM
Real-World Boards Cases ,PULMCCM
Real-World Boards Cases ,PULMCCM
Real-World Boards Cases ,PULMCCM
Real-World Boards Cases ,PULMCCM
Real-World Boards Cases ,PULMCCM
Real-World Boards Cases ,PULMCCM
Real-World Boards Cases ,PULMCCM
Real-World Boards Cases ,PULMCCM
Real-World Boards Cases ,PULMCCM
Real-World Boards Cases ,PULMCCM
Real-World Boards Cases ,PULMCCM

More Related Content

What's hot

Radiological signs in chest medicine Part 1
Radiological signs in chest medicine Part 1Radiological signs in chest medicine Part 1
Radiological signs in chest medicine Part 1Gamal Agmy
 
Diagnostic Imaging of Diffuse Lung Lesions
Diagnostic Imaging of Diffuse Lung LesionsDiagnostic Imaging of Diffuse Lung Lesions
Diagnostic Imaging of Diffuse Lung Lesions
Mohamed M.A. Zaitoun
 
Collapse & consolidation made simple - chest X-rayz
Collapse & consolidation made simple - chest X-rayzCollapse & consolidation made simple - chest X-rayz
Collapse & consolidation made simple - chest X-rayzDrNikrish Hegde
 
Interstitial lung diseases radiology
Interstitial lung diseases radiologyInterstitial lung diseases radiology
Interstitial lung diseases radiology
Shrikant Nagare
 
Hypersensitivity pneumonitis: radiology and pathology aspect
Hypersensitivity pneumonitis: radiology and pathology aspectHypersensitivity pneumonitis: radiology and pathology aspect
Hypersensitivity pneumonitis: radiology and pathology aspect
Thorsang Chayovan
 
Chest imaging of ntm pulmonary disease2
Chest imaging of ntm pulmonary disease2Chest imaging of ntm pulmonary disease2
Chest imaging of ntm pulmonary disease2Choying Chen
 
Approach To Diffuse Parenchymal Lung Diseases
Approach To  Diffuse Parenchymal Lung DiseasesApproach To  Diffuse Parenchymal Lung Diseases
Approach To Diffuse Parenchymal Lung Diseases
Gamal Agmy
 
Presentation1.pptx, radiological signs in thoracic radiology.
Presentation1.pptx, radiological signs in thoracic radiology.Presentation1.pptx, radiological signs in thoracic radiology.
Presentation1.pptx, radiological signs in thoracic radiology.
Abdellah Nazeer
 
Presentation1.pptx, radiological imaging of sarcoidosis.
Presentation1.pptx, radiological imaging of sarcoidosis.Presentation1.pptx, radiological imaging of sarcoidosis.
Presentation1.pptx, radiological imaging of sarcoidosis.
Abdellah Nazeer
 
Typical and Atypical Presentations of Sarcoidosis
Typical and Atypical Presentations of SarcoidosisTypical and Atypical Presentations of Sarcoidosis
Typical and Atypical Presentations of SarcoidosisGamal Agmy
 
HRCT chest Ground glass opacities
HRCT chest Ground glass opacitiesHRCT chest Ground glass opacities
HRCT chest Ground glass opacities
Mitusha Verma
 
Mediastinal lymph nodes
Mediastinal lymph nodesMediastinal lymph nodes
Mediastinal lymph nodes
nipiv17
 
Cavitatory lesions of the lung
Cavitatory lesions of the lungCavitatory lesions of the lung
Cavitatory lesions of the lung
reddyvjm
 
Diffuse parenchymal lung diseases (Postgraduate course)
Diffuse parenchymal lung diseases (Postgraduate course)Diffuse parenchymal lung diseases (Postgraduate course)
Diffuse parenchymal lung diseases (Postgraduate course)
Gamal Agmy
 
Role of hrct in interstitial lung diseases pk upload
Role of hrct in interstitial lung diseases pk uploadRole of hrct in interstitial lung diseases pk upload
Role of hrct in interstitial lung diseases pk upload
Dr pradeep Kumar
 
Pulmonary Tuberculosis - 1
Pulmonary Tuberculosis - 1Pulmonary Tuberculosis - 1
Pulmonary Tuberculosis - 1
Wong Soon Li
 
Diffuse Parenchymal Lung Disease : Radiological Approach
Diffuse Parenchymal Lung Disease : Radiological ApproachDiffuse Parenchymal Lung Disease : Radiological Approach
Diffuse Parenchymal Lung Disease : Radiological Approach
Muhammad Shoyab
 
Electromagnetic Navigation Bronchoscopy (ENB): Clinical Review
Electromagnetic Navigation Bronchoscopy (ENB): Clinical ReviewElectromagnetic Navigation Bronchoscopy (ENB): Clinical Review
Electromagnetic Navigation Bronchoscopy (ENB): Clinical Review
Bassel Ericsoussi, MD
 
Lung Ultrasound Basics
Lung Ultrasound Basics Lung Ultrasound Basics
Lung Ultrasound Basics
Mohammed Fathi El Bagalaty
 

What's hot (20)

Radiological signs in chest medicine Part 1
Radiological signs in chest medicine Part 1Radiological signs in chest medicine Part 1
Radiological signs in chest medicine Part 1
 
Diagnostic Imaging of Diffuse Lung Lesions
Diagnostic Imaging of Diffuse Lung LesionsDiagnostic Imaging of Diffuse Lung Lesions
Diagnostic Imaging of Diffuse Lung Lesions
 
Collapse & consolidation made simple - chest X-rayz
Collapse & consolidation made simple - chest X-rayzCollapse & consolidation made simple - chest X-rayz
Collapse & consolidation made simple - chest X-rayz
 
Interstitial lung diseases radiology
Interstitial lung diseases radiologyInterstitial lung diseases radiology
Interstitial lung diseases radiology
 
Hypersensitivity pneumonitis: radiology and pathology aspect
Hypersensitivity pneumonitis: radiology and pathology aspectHypersensitivity pneumonitis: radiology and pathology aspect
Hypersensitivity pneumonitis: radiology and pathology aspect
 
Chest imaging of ntm pulmonary disease2
Chest imaging of ntm pulmonary disease2Chest imaging of ntm pulmonary disease2
Chest imaging of ntm pulmonary disease2
 
Approach To Diffuse Parenchymal Lung Diseases
Approach To  Diffuse Parenchymal Lung DiseasesApproach To  Diffuse Parenchymal Lung Diseases
Approach To Diffuse Parenchymal Lung Diseases
 
Presentation1.pptx, radiological signs in thoracic radiology.
Presentation1.pptx, radiological signs in thoracic radiology.Presentation1.pptx, radiological signs in thoracic radiology.
Presentation1.pptx, radiological signs in thoracic radiology.
 
Presentation1.pptx, radiological imaging of sarcoidosis.
Presentation1.pptx, radiological imaging of sarcoidosis.Presentation1.pptx, radiological imaging of sarcoidosis.
Presentation1.pptx, radiological imaging of sarcoidosis.
 
Typical and Atypical Presentations of Sarcoidosis
Typical and Atypical Presentations of SarcoidosisTypical and Atypical Presentations of Sarcoidosis
Typical and Atypical Presentations of Sarcoidosis
 
HRCT chest Ground glass opacities
HRCT chest Ground glass opacitiesHRCT chest Ground glass opacities
HRCT chest Ground glass opacities
 
Mediastinal lymph nodes
Mediastinal lymph nodesMediastinal lymph nodes
Mediastinal lymph nodes
 
Cavitatory lesions of the lung
Cavitatory lesions of the lungCavitatory lesions of the lung
Cavitatory lesions of the lung
 
Diffuse parenchymal lung diseases (Postgraduate course)
Diffuse parenchymal lung diseases (Postgraduate course)Diffuse parenchymal lung diseases (Postgraduate course)
Diffuse parenchymal lung diseases (Postgraduate course)
 
Role of hrct in interstitial lung diseases pk upload
Role of hrct in interstitial lung diseases pk uploadRole of hrct in interstitial lung diseases pk upload
Role of hrct in interstitial lung diseases pk upload
 
Pulmonary Tuberculosis - 1
Pulmonary Tuberculosis - 1Pulmonary Tuberculosis - 1
Pulmonary Tuberculosis - 1
 
Diffuse Parenchymal Lung Disease : Radiological Approach
Diffuse Parenchymal Lung Disease : Radiological ApproachDiffuse Parenchymal Lung Disease : Radiological Approach
Diffuse Parenchymal Lung Disease : Radiological Approach
 
Imaging: Endobronchial TB
Imaging: Endobronchial TBImaging: Endobronchial TB
Imaging: Endobronchial TB
 
Electromagnetic Navigation Bronchoscopy (ENB): Clinical Review
Electromagnetic Navigation Bronchoscopy (ENB): Clinical ReviewElectromagnetic Navigation Bronchoscopy (ENB): Clinical Review
Electromagnetic Navigation Bronchoscopy (ENB): Clinical Review
 
Lung Ultrasound Basics
Lung Ultrasound Basics Lung Ultrasound Basics
Lung Ultrasound Basics
 

Similar to Real-World Boards Cases ,PULMCCM

Hoang's pulm review
Hoang's pulm reviewHoang's pulm review
Hoang's pulm review
Chelsea Elise
 
Approach to uawo
Approach to uawoApproach to uawo
Approach to uawo
Raddaa
 
Pneumoniatutor 180316220436
Pneumoniatutor 180316220436Pneumoniatutor 180316220436
Pneumoniatutor 180316220436
JohnCooper855841
 
Pneumonia cases
Pneumonia casesPneumonia cases
Pneumonia cases
Sameh Abdel-ghany
 
Brain abscess
Brain abscessBrain abscess
Brain abscess
Yousuf Shaikh
 
Respirtory part2
Respirtory part2Respirtory part2
Respirtory part2lam808
 
Drs. Olson’s and Jackson’s CMC Pediatric X-Ray Mastery: May Cases
Drs. Olson’s and Jackson’s CMC Pediatric X-Ray Mastery: May CasesDrs. Olson’s and Jackson’s CMC Pediatric X-Ray Mastery: May Cases
Drs. Olson’s and Jackson’s CMC Pediatric X-Ray Mastery: May Cases
Sean M. Fox
 
GR 6 MUMPS AND NCROUPS.pptx2222222222222
GR 6 MUMPS AND NCROUPS.pptx2222222222222GR 6 MUMPS AND NCROUPS.pptx2222222222222
GR 6 MUMPS AND NCROUPS.pptx2222222222222
KelfalaHassanDawoh
 
Rheumatic fever - all you need to know
Rheumatic fever - all you need to knowRheumatic fever - all you need to know
Rheumatic fever - all you need to know
Sid Kaithakkoden
 
Croup
CroupCroup
Pneumonia summary
Pneumonia summaryPneumonia summary
Pneumonia summary
Catherine Par
 
Bronchiectasis
BronchiectasisBronchiectasis
Bronchiectasis
Yatheendra Vasanth
 
Pneumonia.docx
Pneumonia.docxPneumonia.docx
Pneumonia.docx
AreejMohamed21
 
Pneumonia.docx
Pneumonia.docxPneumonia.docx
Pneumonia.docx
AreejMohamed21
 
Diagnosis & Mangement of Community-Acquired Pneumonia, Hospital Acquired Pneu...
Diagnosis & Mangement of Community-Acquired Pneumonia, Hospital Acquired Pneu...Diagnosis & Mangement of Community-Acquired Pneumonia, Hospital Acquired Pneu...
Diagnosis & Mangement of Community-Acquired Pneumonia, Hospital Acquired Pneu...
Riaz Rahman
 
Peds2018 Review PA
Peds2018 Review PA Peds2018 Review PA
Peds2018 Review PA
scott french
 

Similar to Real-World Boards Cases ,PULMCCM (20)

Hoang's pulm review
Hoang's pulm reviewHoang's pulm review
Hoang's pulm review
 
Approach to uawo
Approach to uawoApproach to uawo
Approach to uawo
 
Pneumoniatutor 180316220436
Pneumoniatutor 180316220436Pneumoniatutor 180316220436
Pneumoniatutor 180316220436
 
Croup.ppt
Croup.pptCroup.ppt
Croup.ppt
 
Pneumonia cases
Pneumonia casesPneumonia cases
Pneumonia cases
 
Bela
BelaBela
Bela
 
Brain abscess
Brain abscessBrain abscess
Brain abscess
 
Respirtory part2
Respirtory part2Respirtory part2
Respirtory part2
 
Drs. Olson’s and Jackson’s CMC Pediatric X-Ray Mastery: May Cases
Drs. Olson’s and Jackson’s CMC Pediatric X-Ray Mastery: May CasesDrs. Olson’s and Jackson’s CMC Pediatric X-Ray Mastery: May Cases
Drs. Olson’s and Jackson’s CMC Pediatric X-Ray Mastery: May Cases
 
GR 6 MUMPS AND NCROUPS.pptx2222222222222
GR 6 MUMPS AND NCROUPS.pptx2222222222222GR 6 MUMPS AND NCROUPS.pptx2222222222222
GR 6 MUMPS AND NCROUPS.pptx2222222222222
 
Rheumatic fever - all you need to know
Rheumatic fever - all you need to knowRheumatic fever - all you need to know
Rheumatic fever - all you need to know
 
Croup
CroupCroup
Croup
 
Pneumonia summary
Pneumonia summaryPneumonia summary
Pneumonia summary
 
Bronchiectasis
BronchiectasisBronchiectasis
Bronchiectasis
 
April 24th ppt
April 24th pptApril 24th ppt
April 24th ppt
 
April 24th ppt
April 24th pptApril 24th ppt
April 24th ppt
 
Pneumonia.docx
Pneumonia.docxPneumonia.docx
Pneumonia.docx
 
Pneumonia.docx
Pneumonia.docxPneumonia.docx
Pneumonia.docx
 
Diagnosis & Mangement of Community-Acquired Pneumonia, Hospital Acquired Pneu...
Diagnosis & Mangement of Community-Acquired Pneumonia, Hospital Acquired Pneu...Diagnosis & Mangement of Community-Acquired Pneumonia, Hospital Acquired Pneu...
Diagnosis & Mangement of Community-Acquired Pneumonia, Hospital Acquired Pneu...
 
Peds2018 Review PA
Peds2018 Review PA Peds2018 Review PA
Peds2018 Review PA
 

Recently uploaded

What Are Homeopathic Treatments for Migraines.pdf
What Are Homeopathic Treatments for Migraines.pdfWhat Are Homeopathic Treatments for Migraines.pdf
What Are Homeopathic Treatments for Migraines.pdf
Dharma Homoeopathy
 
CHAPTER 1 SEMESTER V PREVENTIVE-PEDIATRICS.pdf
CHAPTER 1 SEMESTER V PREVENTIVE-PEDIATRICS.pdfCHAPTER 1 SEMESTER V PREVENTIVE-PEDIATRICS.pdf
CHAPTER 1 SEMESTER V PREVENTIVE-PEDIATRICS.pdf
Sachin Sharma
 
CHAPTER 1 SEMESTER V - ROLE OF PEADIATRIC NURSE.pdf
CHAPTER 1 SEMESTER V - ROLE OF PEADIATRIC NURSE.pdfCHAPTER 1 SEMESTER V - ROLE OF PEADIATRIC NURSE.pdf
CHAPTER 1 SEMESTER V - ROLE OF PEADIATRIC NURSE.pdf
Sachin Sharma
 
QA Paediatric dentistry department, Hospital Melaka 2020
QA Paediatric dentistry department, Hospital Melaka 2020QA Paediatric dentistry department, Hospital Melaka 2020
QA Paediatric dentistry department, Hospital Melaka 2020
Azreen Aj
 
Dimensions of Healthcare Quality
Dimensions of Healthcare QualityDimensions of Healthcare Quality
Dimensions of Healthcare Quality
Naeemshahzad51
 
R3 Stem Cells and Kidney Repair A New Horizon in Nephrology.pptx
R3 Stem Cells and Kidney Repair A New Horizon in Nephrology.pptxR3 Stem Cells and Kidney Repair A New Horizon in Nephrology.pptx
R3 Stem Cells and Kidney Repair A New Horizon in Nephrology.pptx
R3 Stem Cell
 
GLOBAL WARMING BY PRIYA BHOJWANI @..pptx
GLOBAL WARMING BY PRIYA BHOJWANI @..pptxGLOBAL WARMING BY PRIYA BHOJWANI @..pptx
GLOBAL WARMING BY PRIYA BHOJWANI @..pptx
priyabhojwani1200
 
Medical Technology Tackles New Health Care Demand - Research Report - March 2...
Medical Technology Tackles New Health Care Demand - Research Report - March 2...Medical Technology Tackles New Health Care Demand - Research Report - March 2...
Medical Technology Tackles New Health Care Demand - Research Report - March 2...
pchutichetpong
 
Navigating Challenges: Mental Health, Legislation, and the Prison System in B...
Navigating Challenges: Mental Health, Legislation, and the Prison System in B...Navigating Challenges: Mental Health, Legislation, and the Prison System in B...
Navigating Challenges: Mental Health, Legislation, and the Prison System in B...
Guillermo Rivera
 
VVIP Dehradun Girls 9719300533 Heat-bake { Dehradun } Genteel ℂall Serviℂe By...
VVIP Dehradun Girls 9719300533 Heat-bake { Dehradun } Genteel ℂall Serviℂe By...VVIP Dehradun Girls 9719300533 Heat-bake { Dehradun } Genteel ℂall Serviℂe By...
VVIP Dehradun Girls 9719300533 Heat-bake { Dehradun } Genteel ℂall Serviℂe By...
rajkumar669520
 
Antibiotic Stewardship by Anushri Srivastava.pptx
Antibiotic Stewardship by Anushri Srivastava.pptxAntibiotic Stewardship by Anushri Srivastava.pptx
Antibiotic Stewardship by Anushri Srivastava.pptx
AnushriSrivastav
 
Surgery-Mini-OSCE-All-Past-Years-Questions-Modified.
Surgery-Mini-OSCE-All-Past-Years-Questions-Modified.Surgery-Mini-OSCE-All-Past-Years-Questions-Modified.
Surgery-Mini-OSCE-All-Past-Years-Questions-Modified.
preciousstephanie75
 
Artificial Intelligence to Optimize Cardiovascular Therapy
Artificial Intelligence to Optimize Cardiovascular TherapyArtificial Intelligence to Optimize Cardiovascular Therapy
Artificial Intelligence to Optimize Cardiovascular Therapy
Iris Thiele Isip-Tan
 
Dehradun ❤CALL Girls 8901183002 ❤ℂall Girls IN Dehradun ESCORT SERVICE❤
Dehradun ❤CALL Girls  8901183002 ❤ℂall  Girls IN Dehradun ESCORT SERVICE❤Dehradun ❤CALL Girls  8901183002 ❤ℂall  Girls IN Dehradun ESCORT SERVICE❤
Dehradun ❤CALL Girls 8901183002 ❤ℂall Girls IN Dehradun ESCORT SERVICE❤
aunty1x2
 
India Clinical Trials Market: Industry Size and Growth Trends [2030] Analyzed...
India Clinical Trials Market: Industry Size and Growth Trends [2030] Analyzed...India Clinical Trials Market: Industry Size and Growth Trends [2030] Analyzed...
India Clinical Trials Market: Industry Size and Growth Trends [2030] Analyzed...
Kumar Satyam
 
Preventing Pickleball Injuries & Treatment
Preventing Pickleball Injuries & TreatmentPreventing Pickleball Injuries & Treatment
Preventing Pickleball Injuries & Treatment
LAB Sports Therapy
 
Global launch of the Healthy Ageing and Prevention Index 2nd wave – alongside...
Global launch of the Healthy Ageing and Prevention Index 2nd wave – alongside...Global launch of the Healthy Ageing and Prevention Index 2nd wave – alongside...
Global launch of the Healthy Ageing and Prevention Index 2nd wave – alongside...
ILC- UK
 
GURGAON Call Girls ❤8901183002❤ #ℂALL# #gIRLS# In GURGAON ₹,2500 Cash Payment...
GURGAON Call Girls ❤8901183002❤ #ℂALL# #gIRLS# In GURGAON ₹,2500 Cash Payment...GURGAON Call Girls ❤8901183002❤ #ℂALL# #gIRLS# In GURGAON ₹,2500 Cash Payment...
GURGAON Call Girls ❤8901183002❤ #ℂALL# #gIRLS# In GURGAON ₹,2500 Cash Payment...
ranishasharma67
 
Jaipur ❤cALL gIRLS 89O1183002 ❤ℂall Girls IN JaiPuR ESCORT SERVICE
Jaipur ❤cALL gIRLS 89O1183002 ❤ℂall Girls IN JaiPuR ESCORT SERVICEJaipur ❤cALL gIRLS 89O1183002 ❤ℂall Girls IN JaiPuR ESCORT SERVICE
Jaipur ❤cALL gIRLS 89O1183002 ❤ℂall Girls IN JaiPuR ESCORT SERVICE
ranishasharma67
 
Overcome Your Phobias with Hypnotherapy.pptx
Overcome Your Phobias with Hypnotherapy.pptxOvercome Your Phobias with Hypnotherapy.pptx
Overcome Your Phobias with Hypnotherapy.pptx
renewlifehypnosis
 

Recently uploaded (20)

What Are Homeopathic Treatments for Migraines.pdf
What Are Homeopathic Treatments for Migraines.pdfWhat Are Homeopathic Treatments for Migraines.pdf
What Are Homeopathic Treatments for Migraines.pdf
 
CHAPTER 1 SEMESTER V PREVENTIVE-PEDIATRICS.pdf
CHAPTER 1 SEMESTER V PREVENTIVE-PEDIATRICS.pdfCHAPTER 1 SEMESTER V PREVENTIVE-PEDIATRICS.pdf
CHAPTER 1 SEMESTER V PREVENTIVE-PEDIATRICS.pdf
 
CHAPTER 1 SEMESTER V - ROLE OF PEADIATRIC NURSE.pdf
CHAPTER 1 SEMESTER V - ROLE OF PEADIATRIC NURSE.pdfCHAPTER 1 SEMESTER V - ROLE OF PEADIATRIC NURSE.pdf
CHAPTER 1 SEMESTER V - ROLE OF PEADIATRIC NURSE.pdf
 
QA Paediatric dentistry department, Hospital Melaka 2020
QA Paediatric dentistry department, Hospital Melaka 2020QA Paediatric dentistry department, Hospital Melaka 2020
QA Paediatric dentistry department, Hospital Melaka 2020
 
Dimensions of Healthcare Quality
Dimensions of Healthcare QualityDimensions of Healthcare Quality
Dimensions of Healthcare Quality
 
R3 Stem Cells and Kidney Repair A New Horizon in Nephrology.pptx
R3 Stem Cells and Kidney Repair A New Horizon in Nephrology.pptxR3 Stem Cells and Kidney Repair A New Horizon in Nephrology.pptx
R3 Stem Cells and Kidney Repair A New Horizon in Nephrology.pptx
 
GLOBAL WARMING BY PRIYA BHOJWANI @..pptx
GLOBAL WARMING BY PRIYA BHOJWANI @..pptxGLOBAL WARMING BY PRIYA BHOJWANI @..pptx
GLOBAL WARMING BY PRIYA BHOJWANI @..pptx
 
Medical Technology Tackles New Health Care Demand - Research Report - March 2...
Medical Technology Tackles New Health Care Demand - Research Report - March 2...Medical Technology Tackles New Health Care Demand - Research Report - March 2...
Medical Technology Tackles New Health Care Demand - Research Report - March 2...
 
Navigating Challenges: Mental Health, Legislation, and the Prison System in B...
Navigating Challenges: Mental Health, Legislation, and the Prison System in B...Navigating Challenges: Mental Health, Legislation, and the Prison System in B...
Navigating Challenges: Mental Health, Legislation, and the Prison System in B...
 
VVIP Dehradun Girls 9719300533 Heat-bake { Dehradun } Genteel ℂall Serviℂe By...
VVIP Dehradun Girls 9719300533 Heat-bake { Dehradun } Genteel ℂall Serviℂe By...VVIP Dehradun Girls 9719300533 Heat-bake { Dehradun } Genteel ℂall Serviℂe By...
VVIP Dehradun Girls 9719300533 Heat-bake { Dehradun } Genteel ℂall Serviℂe By...
 
Antibiotic Stewardship by Anushri Srivastava.pptx
Antibiotic Stewardship by Anushri Srivastava.pptxAntibiotic Stewardship by Anushri Srivastava.pptx
Antibiotic Stewardship by Anushri Srivastava.pptx
 
Surgery-Mini-OSCE-All-Past-Years-Questions-Modified.
Surgery-Mini-OSCE-All-Past-Years-Questions-Modified.Surgery-Mini-OSCE-All-Past-Years-Questions-Modified.
Surgery-Mini-OSCE-All-Past-Years-Questions-Modified.
 
Artificial Intelligence to Optimize Cardiovascular Therapy
Artificial Intelligence to Optimize Cardiovascular TherapyArtificial Intelligence to Optimize Cardiovascular Therapy
Artificial Intelligence to Optimize Cardiovascular Therapy
 
Dehradun ❤CALL Girls 8901183002 ❤ℂall Girls IN Dehradun ESCORT SERVICE❤
Dehradun ❤CALL Girls  8901183002 ❤ℂall  Girls IN Dehradun ESCORT SERVICE❤Dehradun ❤CALL Girls  8901183002 ❤ℂall  Girls IN Dehradun ESCORT SERVICE❤
Dehradun ❤CALL Girls 8901183002 ❤ℂall Girls IN Dehradun ESCORT SERVICE❤
 
India Clinical Trials Market: Industry Size and Growth Trends [2030] Analyzed...
India Clinical Trials Market: Industry Size and Growth Trends [2030] Analyzed...India Clinical Trials Market: Industry Size and Growth Trends [2030] Analyzed...
India Clinical Trials Market: Industry Size and Growth Trends [2030] Analyzed...
 
Preventing Pickleball Injuries & Treatment
Preventing Pickleball Injuries & TreatmentPreventing Pickleball Injuries & Treatment
Preventing Pickleball Injuries & Treatment
 
Global launch of the Healthy Ageing and Prevention Index 2nd wave – alongside...
Global launch of the Healthy Ageing and Prevention Index 2nd wave – alongside...Global launch of the Healthy Ageing and Prevention Index 2nd wave – alongside...
Global launch of the Healthy Ageing and Prevention Index 2nd wave – alongside...
 
GURGAON Call Girls ❤8901183002❤ #ℂALL# #gIRLS# In GURGAON ₹,2500 Cash Payment...
GURGAON Call Girls ❤8901183002❤ #ℂALL# #gIRLS# In GURGAON ₹,2500 Cash Payment...GURGAON Call Girls ❤8901183002❤ #ℂALL# #gIRLS# In GURGAON ₹,2500 Cash Payment...
GURGAON Call Girls ❤8901183002❤ #ℂALL# #gIRLS# In GURGAON ₹,2500 Cash Payment...
 
Jaipur ❤cALL gIRLS 89O1183002 ❤ℂall Girls IN JaiPuR ESCORT SERVICE
Jaipur ❤cALL gIRLS 89O1183002 ❤ℂall Girls IN JaiPuR ESCORT SERVICEJaipur ❤cALL gIRLS 89O1183002 ❤ℂall Girls IN JaiPuR ESCORT SERVICE
Jaipur ❤cALL gIRLS 89O1183002 ❤ℂall Girls IN JaiPuR ESCORT SERVICE
 
Overcome Your Phobias with Hypnotherapy.pptx
Overcome Your Phobias with Hypnotherapy.pptxOvercome Your Phobias with Hypnotherapy.pptx
Overcome Your Phobias with Hypnotherapy.pptx
 

Real-World Boards Cases ,PULMCCM

  • 1. 1 Real-WorldBoards Question 1 of 82 For about 2 weeks, initially with nasal congestion, rhinorrhea and a sore throat, now with a steady cough productive of yellow sputum. She feels better except for the cough, which is impairing her functioning at work. Her temperatures (taken at home) have all been under 101 F. You notice she seems a bit under the weather but is breathing normally; you take her radial pulse, which is 89. She asks you what she should buy for her cough. What's the best recommendation for her, based on available evidence? A. Cetirizine for 7 days B. Brompheniramine with pseudoephedrine, and naprosyn, until symptoms subside C. Azithromycin for 5 days D. Chicken soup E. Schedule an office visit with chest films You answered. We feelthat B and D are the best answers. Acute cough (< 3 weeks) is believed to be most often due to the common cold. Cough can be persistent and bothersome, justifying treatment. One randomized trial showed improvement in cough with the use of brompheniramine (a first generation antihistamine) with pseudoephedrine. Naprosyn bid also reduced cough (and other cold symptoms)in a randomized trial. These two therapies are "A"
  • 2. 2 recommendations by the ACCP for patients with bothersome cough due to the common cold. Loratadine with pseudoephedrine was no better than placebo in another study; other studies also show that newer-generation nonsedating antihistamines are not effective on cold symptoms. A systematic review concluded that in fact, no over-the-counter remedies are effective for cough from the common cold; however, it lumped together new antihistamines (ineffective) and older ones (possibly effective) and concluded antihistamines are not effective. It also doubted whether the benefits seen in positive trials were clinically significant. Reassurance seems a reasonable option, too. Office visits, chest films, and antibiotics are not needed for healthy people with colds. Fever, immune suppression, tachycardia, or tachypnea might prompt a chest X-ray. Also: Acute bacterial sinusitis can present with postnasal drip cough and upper respiratory tract infection, and be indistinguishable from the common cold (even on sinus imaging). Cough that persists > 3 weeks should be evaluated as chronic cough. Pertussis causes persistent cough, is underdiagnosed, and should be treated with macrolide antibiotics. The CDC recommends pertussis boostervaccination for all adults. Question 2 of 82 (score?) This 78 year old white woman has been coughing for "a good while"; you estimate at least many months by her history. She is persistently fatigued, has lost 5 pounds, but has no fevers or night sweats. She is a never-smoker. She sometimes coughs up blood-streaked sputum. She lives in a high-rise retirement home. Other cuts of her chest CT are similar to the one below, correlating well with the findings on chest X-ray.
  • 3. 3
  • 4. 4 The primary problem is likely: A. An infection she caught from the environment B. An infection she caught from someone else C. Reactivation tuberculosis D. Bronchioloalveolar carcinoma E. A chronic process resulting from repeated past infections We feelthat A is the best answer. This is a mycobacterium avium complex infection, with Lady Windemere syndrome (bronchiectasis and multiple small nodules often involving the right middle lobe or lingula in nonsmoking elderly women, often with chest wall or skeletal deformities). The other options are possible but the pattern on imaging along with the history are most suggestive of MAC, an endemic organism inhaled
  • 5. 5 from soil or water. Her bronchiectasis could perpetuate her cough and hemoptysis; however, she also has many nodules that are indicative of ongoing infection as the primary process.Nontuberculous mycobacteria (NTM) also include M. kansasii, rapidly growing mycobacteria (RGM), and numerous less common species. Nonpathogenic airway colonization by these organisms is frequently incidentally discovered and must be differentiated from infection. NTM infection commonly causes a TB-like pattern (upper lobe cavitary disease); hypersensitivity pneumonitis (ground glass and centrilobular nodules, sometimes with a mosaic pattern), or the pattern seen here (nodular bronchiectasis with or without fibrocavitary disease), along with low-grade nonspecific symptoms usually including cough and fatigue. Diagnosis requires compatible imaging and clinical findings, along with 2 positive sputum samples (24+ hrs apart) or 1 positive sample from bronchoscopic lavage or biopsy. This unfortunate woman has required more than 3 years of multi drug therapy, with repeated medication switches for adverse effects including optic neuropathy while on ethambutol--still, her sputum remains positive for MAC. Unfortunately, her situation is not atypical for people with this indolent, difficult- to-treat infection. National Jewish in Denver are the premier referral / consultation center for these cases. Question 3 of 82 (score?) You receive an email from a colleague asking for a "curbsideconsult." You haven't heard the story or your colleague's question yet, but to frame your thinking, you quickly glance at the images she's attached to the email.
  • 6. 6
  • 7. 7
  • 8. 8 Based on these images, what's the most likely clinical scenario? A. A 50-year old man with a mass in the head of the pancreas. B. A 66-year old man with progressive dyspnea on exertion, leg edema, and longstanding hypertension. C. A 48-year old woman treated for malignant melanoma 2 years ago. D. A 24-year old woman with HIV-AIDS, 4 weeks of dyspnea and dry cough, SaO2 of 88% on ambient air, non-adherent with Bactrim prophylaxis. E. A 60-year old chicken farmer with progressive dyspnea and a positive hypersensitivity panel. We feelthat A is the best answer. The chest X-ray shows reticulonodular opacities, a nonspecific abnormal finding. The axial CT cuts show:  Small nodules in a lymphangitic pattern, i.e., touching the pleura/fissures  Pleural effusions with an irregular, nodular contour
  • 9. 9  Irregular thickening of the interlobular septa  Peribronchovascular thickening All these findings are consistent with this patient's diagnosis of lymphangitic carcinomatosis(LC): diffuse metastatic spread of cancer to the lungs, where it diffusely infiltrates and obstructs lymphatic channels. 80% of cases are due to adenocarcinomas, most often from a primary cancer of the breast, lung, or colon. This case of LC was from a primary pancreatic adenocarcinoma. Chest films are usually normal in lymphangitic carcinomatosis. This patient's imaging studies are also consistent with:  Sarcoidosis (would expect hilar lymphadenopathy)  Lymphoma  Silicosis  Coal worker's pneumoconiosis Regarding the other answer choices, all but metastatic melanoma would likely include reticular opacities on chest X-ray. Their patterns on CT imaging would most likely include:  Cardiogenic pulmonary edema (CHF): Bilateral/diffuse interlobular and intralobular septal thickening, ground glass opacities, pleural effusions. Centrilobular nodules (ill-defined/hazy) may be seen, surrounded by ground glass opacity.  Metastatic melanoma: Nodules/masses, usually multiple, most often >1-2 cm, in a random pattern affecting one or both lungs. Diffuse lymphangitic involvement would be highly unusual.  Pneumocystis pneumonia: Bilateral/diffuse ground glass opacities (90%); nodules are rare.  Hypersensitivity pneumonitis: Bilateral/diffuse centrilobular nodules, surrounded by ground glass opacities, often with tree-in-bud opacities. Later in the illness, fibrosis (traction bronchiectasis, honeycombing) is present. Nodule mini-primer:Centrilobular nodules appear at the center of the secondary pulmonary lobules; they represent engorgement of the pulmonary arteriole and/or occlusion of the centrilobular bronchiole. Centrilobular nodules do not touch the pleural surfaces or fissures. (Lymphangitic nodules and random nodules may touch the fissures/pleural surfaces.) Lymphangitic nodules are accompanied by irregular thickening of septae and bronchovascular bundles; they occurfrom lymphangitic
  • 10. 10 spread of tumor, sarcoid, or another inflammatory process. Randomnodules result from hematogenous spread of tumor or disseminated infection (tuberculosis, fungal infection) and occurwithout the associated findings of lymphangitically-spread nodules. Question 4 of 82 (score?) Your colleague in the ED calls you. He has a 55-year-old former nurse there, sent by her PCP for leg swelling. He ordered lower extremity ultrasound; the left common femoral vein image is shown. The image on your left is with the probe resting on the skin; the image on the right is with compressionof the vein by the ultrasound probe. She has edema in the left leg. She has no risk factors for or history of DVT. Vital signs are normal. A radiologist will be available to interpret the images in a few hours. After your history and exam reveal nothing else, the ED doc asks what you want to do. Of the listed options, what is the best recommendation you could make? A. Schedule placement of an IVC filter while awaiting the final radiology read. B. Admit; start warfarin and unfractionated heparin; recommend warfarin for 3 months. C. Start enoxaparin and warfarin, then discharge; recommend warfarin indefinitely.
  • 11. 11 D. Get dedicated Doppler ultrasound of the leg veins. E. Pulmonary angiogram. We feelthat C is the best answer. This is a proximal, unprovoked, asymptomatic DVT. The vein should compress completely; this one does not, and has an echogenic area in the lumen (it should be homogenously jet-black inside). Generally speaking, recurrence rate after unprovoked DVT is similar between people with and without identifiable hypercoagulable states. ACCP recommendations for people with unprovoked proximal DVT without PE are that anticoagulation be continued "long-term" (i.e., indefinitely), if the patient is amenable after a risk/benefit discussion(Grade 1A, "Antithrombotic Therapy for VTE Disease," Section 2.1.2). ACCP also recommends warfarin be begun the same day as enoxaparin. There is growing acceptance of treating asymptomatic DVT on an outpatient basis "if possible" (Grade 1C ACCP recommendation), which we take to mean the patient is stable, can self-inject enoxaparin, and close follow-up can be arranged for INR checks. Question 5 of 82 (score?) Your internal medicine colleague asks you about a patient she is about to discharge home after a hospitalization for a COPD exacerbation. The patient, Mr. M., takes a beta-blocker for hypertension. Mr. M. does not have a diagnosis of coronary artery disease. Your colleague is considering stopping the beta-blocker to avoid any contribution to future COPD exacerbations, but wants your opinion first. What do you recommend? A. Stop the beta blocker. B. Continue the beta blocker. C. Stop the beta blocker; order a stress test. D. Continue the beta blocker; order an echocardiogram. We feelthat B is the best answer. Cardioselective beta-blockers are safe in patients with COPD, and may in fact be beneficial, mounting evidence suggests. A cohortstudy published in 2008 among
  • 12. 12 >3,000 people with cardiovascular disease suggested beta blockade is not associated with a reduced FEV1 or increased exacerbations. In an observational database study of almost 6,000 outpatients with COPDpublished in 2011, beta blocker use was associated with reduced mortality and fewer COPDexacerbations. Cardioselective beta-blockers are likely beneficial for patients with mild or moderate asthma, as well, although that data is less conclusive than for COPD. Early case reports of beta blockers precipitating asthma exacerbations were in patients taking noncardioselective beta blockers (e.g., propranolol). Clinical Takeaway:Beta-blockers may be prescribed without restriction to people with COPD, if there is no contraindication. Question 6 of 82 (score?) A 46-year-old woman with persistent asthma presents for her routine 3 month visit. She notes her symptoms have been worse for the past few weeks. She is using albuterol daily, and having cough and wheezing a few nights a week. She has stopped her daily walk for exercise, due to difficulty breathing. Her vital signs are normal. On exam, she is breathing comfortably but has wheezing diffusely. She is taking fluticasone 220 mcg twice daily (medium dose) and albuterol 3 times daily. She demonstrates her inhaler technique, which is correct. What should be the next step? A. Schedule spirometry for next week to guide step-up therapy. B. Increase the doseof fluticasone to 440 mcg b.i.d. C. Provide prednisone 40 mg for 5 days and see again next week. D. Add a long-acting beta agonist. E. Add a leukotriene receptor antagonist.. We agree that B, C, D, and E are all reasonable answers. The patient needs step-up therapy for uncontrolled asthma. Traditionally that would have meant adding a long-acting beta agonist, but the FDA's 2010 warnings
  • 13. 13 about long-acting beta agonists for asthma have introduced uncertainty (or call it flexibility) into clinical decision making for uncontrolled asthma. The National Asthma Education and Prevention Program (NAEPP guidelines) for 2007 would consider her asthma to be either:  "Notwell controlled": Step-up by 1 step; see again in 2-6 weeks; use alternative treatments if side effects develop.  "Very poorly controlled":Consider short course of oral steroids; step-up 1-2 steps;see again in 2 weeks; use alternative agents if side effects develop. Per NAEPP guidelines, step-up therapy would definitely include adding a long- acting beta agonist, and 2 steps up would also include increasing to high doseICS (440 or 500 mcg fluticasone twice daily). NAEPP-recommended alternatives to a LABA include a leukotriene receptor antagonist, Zileuton, theophylline, and at higher steps, omalizumab for those with allergies. Since February 2010, FDA black-box warnings for LABAs in asthma have created uncertainty around these guidelines. The most current language at FDA.gov (June 2010) would supportLABA use in this patient (inadequate control on medium- doseICS). Given the confusion at present, though, it would be hard to argue with a physician who choseto use an alternative agent in place of a LABA, or increase the inhaled corticosteroid dosewithout adding a LABA. Under direction by the FDA, major pharma companies are launching 5 large clinical trials to answer the question of LABA safety for asthma; results are expected in ~2018 or so. Delaying step-up therapy to obtain spirometry does not seem appropriate. Spirometry can help place a patient in a category on the NAEPP algorithm; however, regardless of the test result, step-up therapy is indicated and should be started now. Question 7 of 82 (score?) John, 33, is referred to you by his primary doctorfor bronchiectasis. John has had recurrent respiratory infections since graduating high school. He takes inhaled albuterol as needed, which in practice is never, becausehe does not perceive a benefit. His symptoms of episodic cough, dyspnea, and infections wax and wane unpredictably, but are progressing overall. Exam and vitals reveal distant breath sounds with some rhonchi, and are otherwise normal. A thorough occupational /
  • 14. 14 inhalational history is negative. As you consider beginning an extensive workup for unexplained bronchiectasis, you examine the test results below. Question 8 of 82 (score?) A 74-year-old man with severe, end-stage COPD(FEV1 20% predicted) suffers from breathlessness that is so severe, any activity (even standing from a chair) makes him feel like "I'm gonna die." He is on maximal inhaler therapies and supplemental oxygen. He has tried pulmonary rehabilitation but he could not tolerate it. To reduce this patient's feelings of severe breathlessness, which intervention has the strongest evidence for effectiveness? A. Nebulized morphine sulfate. B. Oral morphine sulfate. C. Using a fan to blow air on his face. D. Biofeedback therapy. We feelthat B is the best answer. Several systematic reviews and meta-analyses of randomized trials supportthe use of opioid use to relieve severe dyspnea in people with end-stage COPD. Many other therapies to relieve dyspnea in COPDhave been tested in randomized trials. Positive results were found from chest wall vibration and neuroelectrical muscle stimulation. Nebulized morphine also did not show an effect in randomized trials. Acupuncture, music therapy, use of a fan, and biofeedback/relaxation showed inconsistent or no benefits. The lack of conclusive evidence in randomized trials does not mean these nonpharmacologic therapies shouldn't be tried, as they may work in individual patients, and a placebo or psychological effect may be beneficial to maximize comfort in a palliative treatment plan, especially with therapies that are safe and simple.
  • 15. 15
  • 16. 16
  • 17. 17 What's the diagnosis? A. Williams-Campbell syndrome. B. Allergic bronchopulmonary aspergillosis. C. Mounier-Kuhn syndrome.
  • 18. 18 D. Cystic fibrosis. E. Marfan syndrome. We feelthat C is the best answer. Mounier-Kuhn syndrome, or congenital tracheobronchomegaly, is a rare inherited disorder of cartilage formation resulting in enlargement of the C-rings in the segmental bronchi and trachea. More distal bronchial structures are normal. (This is in contradistinction to Williams-Campbell syndrome, in which central airways are normal but distal airways are dilated.) The diagnosis is readily made on a CT scan showing grossly enlarged central airways, especially with compatible symptoms. Clinical presentation is usually with recurrent lower respiratory infections, which may begin in childhood, or not until young adulthood. Diagnosis was delayed until age ~75 in a few cases. Productive cough, dyspnea, and poorclearance of secretions are common symptoms. Prognosis varies widely and is impossible to accurately predict. Progressive bronchiectasis (perhaps due to repeated infections), emphysema, and pulmonary fibrosis are possible. Mild, stable cases, as well as those progressing to respiratory failure and death, have been reported. Due to the condition's rarity, no evidence is available to guide management, but airway hygiene has been recommended: postural drainage and consideration of other airway clearance techniques (chest physiotherapy, flutter valves, bronchodilators, dornasealpha, etc). Positive-airway pressure and airway stenting have been proposed. Surgery (posterior membranous tracheobronchoplasty) may be helpful in certain severely affected patients, but is usually not technically feasible. The cartilage defect in Marfan syndrome can cause tracheobronchomalacia, and either Marfan, cystic fibrosis, or ABPA can cause bronchiectasis with cystic degeneration of the lungs, but none of these cause tracheobronchomegaly.
  • 19. 19 Question 8 of 82 (score?) A 74-year-old man with severe, end-stage COPD(FEV1 20% predicted) suffers from breathlessness that is so severe, any activity (even standing from a chair) makes him feel like "I'm gonna die." He is on maximal inhaler therapies and supplemental oxygen. He has tried pulmonary rehabilitation but he could not tolerate it. To reduce this patient's feelings of severe breathlessness, which intervention has the strongest evidence for effectiveness? A. Nebulized morphine sulfate. B. Oral morphine sulfate. C. Using a fan to blow air on his face. D. Biofeedback therapy. We feelthat B is the best answer. Several systematic reviews and meta-analyses of randomized trials supportthe use of opioid use to relieve severe dyspnea in people with end-stage COPD. Many other therapies to relieve dyspnea in COPDhave been tested in randomized trials. Positive results were found from chest wall vibration and neuroelectrical muscle stimulation. Nebulized morphine also did not show an effect in randomized trials. Acupuncture, music therapy, use of a fan, and biofeedback/relaxation showed inconsistent or no benefits. The lack of conclusive evidence in randomized trials does not mean these nonpharmacologic therapies shouldn't be tried, as they may work in individual patients, and a placebo or psychological effect may be beneficial to maximize comfort in a palliative treatment plan, especially with therapies that are safe and simple. Question 9 of 82 (score?) This 49 year old U.S. citizen originally from Trinidad is referred to you for evaluation by her oncologist. She had the below chest X-ray after a mammogram suggested abnormal lung parenchyma. Breast biopsy showed ductal carcinoma in situ. She is to have lumpectomy soon. She has been mildly short of breath since her 20s, which has insidiously gotten worse, although she still works full-time and takes care of 3 children. She also has a chronic dry cough worse on deep
  • 20. 20 inspiration. She has never sought evaluation for her symptoms and has never had prior chest imaging. In your office, she walks 1000 feet in 6 minutes, with desaturation from 100% on ambient air to 85% on ambulation with mild dyspnea. A TB skin test is placed. An HIV test was negative last year. PFTs:TLC 70% predicted (below lower limit of normal), FEV1/FVC ratio 0.65, FEV1 2.1L (below lower limit of normal) DLCO 40% predicted, DLCO/VA 80% predicted.
  • 21. 21
  • 22. 22 Without any other information, her most likely diagnosis is: A. Idiopathic pulmonary fibrosis B. Sarcoidosis C. Tuberculosis D. Lymphangitic spread of breast cancer We feelthat B is the best answer.
  • 23. 23 Sarcoidosis is more prevalent in people of African descent, and can present at a young age and progress insidiously. This woman likely has diffuse lung fibrosis without hilar lymphadenopathy, so-called stage IV sarcoid. The term "stage" misleadingly implies a predictable pattern of linear disease progression; in fact, the radiographic pattern (stage) correlates neither with chronicity nor with changes in pulmonary function. Although patients usually have restrictive lung disease at presentation, as many as 50% also have obstructive disease. Bronchodilator responsiveness is not uncommon. Spirometry returns to normal in 80% of patients within 2 years (with about 67% of patients achieving a complete remission of sarcoid within 10 years). Sarcoidosis is a diagnosis of exclusion, with a minimum requirement of a biopsy demonstrating noncaseating granulomas (except in certain patients, such as those with Lofgren's syndrome, Heerfordt's syndrome, or asymptomatic, incidentally discovered hilar lymphadenopathy). At that point, other granulomatous diseases must be excluded. This patient had a noncaseating granuloma on breast biopsy -- which can also be seen in breast cancer and which therefore still left some diagnostic uncertainty. Her symptoms and imaging have been stable over years since her first visit; she remained in remission from breast cancer after lumpectomy and radiation. Miliary TB would be expected to producea more nodular pattern on chest films, as opposedto this linear or reticular pattern. In situ cancer would not likely have spread lymphangitically. IPF usually preferentially affects the peripheral and basal lungs. The small nodule in the right lung baseis probably of no significance. Question 10 of 82 (score?) A 28-year-old medical student approaches you in the hospital hallway. She says with dismay, "My PPD is positive." She holds out her arm, which shows 12 mm of induration marked off by an examiner. You slide the edge of your pen tip along her skin until you meet resistance on each side. You get the same margins as the examiner's. The student says, "I just got a PPD 3 weeks ago that was negative - I know it was. But I got busy and never had it checked." She has no known exposures to tuberculosis, and no respiratory or constitutional symptoms. She had a chest X-ray this morning which was normal.
  • 24. 24 She had a negative TB skin test last year when she first came to the US from Vietnam (a TB-endemic country), where she grew up. She did get BCG vaccine there as a child. You refer her to your hospital's occupational health department for further advice and management. Assuming the skin test 3 weeks ago was truly negative, and the chest X-ray this morning was normal, what would be the best recommendation? A. Repeat skin test next year; this was a boosterreaction. B. Prescribe isoniazid daily for 9 months. C. Repeat skin testing in one month. D. Collect serum for interferon-gamma release assay. E. Repeat skin test next year; this was a false-positive from BCG vaccine. We agree that B is the best answer. The student has a positive tuberculin skin test (TST) confirming latent tuberculosis infection (LTBI), probably occurring years ago in her home country. (Cutoff for TST positivity is 10 mm in health care workers; other cutoffs are here.) She did have a boosterreaction. Boosterreactions are initial false-negatives that become true-positives. They occurwhen a previously-infected personloses hypersensitivity to PPD antigen over time, resulting in a negative TB skin test. However, the injected PPD re-stimulates the immune system, and a TB skin test repeated 1-3 weeks later will induce a positive induration reaction. Because of this, the gold standard for TB skin testing is actually to place 2 PPDs 1-3 weeks apart; if either is positive, the person is considered to have LTBI and treatment should be considered. Treatment for LTBI can reduce the lifetime risk of reactivation tuberculosis by 60-90%:  First-line therapy is 9 months of daily isoniazid (5mg/kg, max 300 mg).  Six months of INH is also acceptable and supported by randomized trial data.  Other acceptable regimens include twice-weekly INH 300 mg for 6 or 9 months, or daily rifampin for 4 to 6 months. Data is less robust for these.
  • 25. 25 Both INH and rifampin have potential toxicity, which must be balanced against the risk of reactivation tuberculosis. There is a 1 in 1000 risk of hepatitis while taking INH for alcohol-abstinent people without preexisting liver disease; asymptomatic transaminitis is common. With monthly monitoring, intermittent dosing, and provision of pyridoxine, INH can be given safely to most patients. Without treatment, the lifetime risk of developing reactivationtuberculosis in HIV-negative people is ~5-10%, with most of that risk in the first 2 years following infection. By CDC guidelines, the other answer choices are not recommended:  BCG-vaccinated people should be tested and treated for LTBI the same as everyone else.  Interferon-gamma release assay and TST should not be routinely performed together. In this case, the patient was tested with the gold standard two-step approach(although accidentally) and should be considered to have LTBI; confirmatory testing isn't needed.  Her skin test would be expected to still be positive in one month. Once a TB skin test is positive once in life, it will always be positive (or falsely negative). There is no way to detect re-infection with tuberculosis with skin testing. Question 11 of 82 (score?) During 25 days in your ICU, Mr. F has survived postoperative sepsis after a ventral hernia repair, and ARDS with prolonged respiratory failure with a tracheostomy. He still requires 20 mmHg of pressure supportventilation overnight, but tolerates trach collar during the days. A local long-term acute care hospital is eager to take over his care, and his transfer paperwork is ready. His wife, Wanda, wants to know what to expect now. Mr F is too weak and confused to communicate meaningfully, but she thinks her husband would want to go on fighting if he had a good chance to survive and get back to a place where he could at least live at home with assistance. According to published literature, what are Mr F's chances of being alive in one year?
  • 26. 26 A. 75% B. 50% C. 25% D. 10% E. It's impossible to predict from the information provided. We agree that B and C are the best answers. Chronic critical illness is an epiphenomenon resulting both from advances in intensive care and the financial success (and related proliferation) of long-term acute care (LTAC) hospitals. Generally defined as respiratory failure lasting weeks, chronic critical illness affects more than 100,000 people in the U.S. (estimated), and costs more than $20 billion annually. What happens to your patient after you send him or her to an LTAC? - Most are not freed from mechanical ventilation (reported weaning rates: 30 - 50%); if success is achieved, it's almost always within 60 days. - More than 40% are readmitted to the hospital within a year. - At the end of one year, fewer than half will be alive (32 - 52%). - Fewer than 12% will be alive and independent 1 year after their acute illness. Surveys of patients' families show that most have no idea of these grim odds. It's uncertain whether that's because physicians fail to effectively communicate the reality of the situation, or whether it's due to "selective hearing" by emotionally overwhelmed families desperate for hope. Although it's true that we can't predict with perfect accuracy what will happen to any individual patient, in the absence of clear reasons to be optimistic it might not be appropriate to say things like "He just needs more time" or "We just can't predict these things," since the data show that we usually can (at least in terms of predicting functional independence after LTAC admission, which is an unlikely outcome
  • 27. 27 What is the bestintravenous medication infusion to provide next? A. Lidocaine. B. Esmolol. C. Heparin. D. Diltiazem. We feelthat C is the best answer. After successfulemergent electrocardioversion of atrial fibrillation, heparin infusion is appropriate if there is no contraindication. It is not known how long the patient was in a-fib before, and a dangerous clot may be present in the left atrium / left atrial appendage, increasing the risk for stroke. Cardiology consultation should also be sought. Diltiazem and beta blocker infusions are not necessary since she is in normal sinus rhythm. Lidocaine is an alternate agent to amiodarone for ventricular tachycardia or fibrillation. Question 12 of 82 (score?) During 25 days in your ICU, Mr. F has survived postoperative sepsis after a ventral hernia repair, and ARDS with prolonged respiratory failure with a tracheostomy. He still requires 20 mmHg of pressure supportventilation overnight, but tolerates trach collar during the days. A local long-term acute care hospital is eager to take over his care, and his transfer paperwork is ready.
  • 28. 28 His wife, Wanda, wants to know what to expect now. Mr F is too weak and confused to communicate meaningfully, but she thinks her husband would want to go on fighting if he had a good chance to survive and get back to a place where he could at least live at home with assistance. According to published literature, what are Mr F's chances of being alive in one year? A. 75% B. 50% C. 25% D. 10% E. It's impossible to predict from the information provided. We agree that B and C are the best answers. Chronic critical illness is an epiphenomenon resulting both from advances in intensive care and the financial success (and related proliferation) of long-term acute care (LTAC) hospitals. Generally defined as respiratory failure lasting weeks, chronic critical illness affects more than 100,000 people in the U.S. (estimated), and costs more than $20 billion annually. What happens to your patient after you send him or her to an LTAC? - Most are not freed from mechanical ventilation (reported weaning rates: 30 - 50%); if success is achieved, it's almost always within 60 days. - More than 40% are readmitted to the hospital within a year. - At the end of one year, fewer than half will be alive (32 - 52%). - Fewer than 12% will be alive and independent 1 year after their acute illness. Surveys of patients' families show that most have no idea of these grim odds. It's uncertain whether that's because physicians fail to effectively communicate the reality of the situation, or whether it's due to "selective hearing" by emotionally overwhelmed families desperate for hope. Although it's true that we can't predict with perfect accuracy what will happen to any individual patient, in the absence of clear reasons to be optimistic it might not be appropriate to say things like "He just needs more time" or "We just can't predict these things," since the data show that
  • 29. 29 we usually can (at least in terms of predicting functional independence after LTAC admission, which is an unlikely outcome Question 13 of 82 (score?) John, 44, comes to see you in your clinic, complaining of worsening exertional dyspnea. He smoked a half a pack a day for 5 years, quitting 25 years ago. He recently quit his construction job becausehe couldn't unload sacks of concrete or walk the site quickly enough to keep up. PFTs:FEV1 1.1 L (35% predicted); FEV1/FVC ratio 0.45; DLCO 35% predicted. Vitals: HR 90, RR 18, BP 125/85, SaO2 94% on ambient air, 92% with ambulation. He saw his primary doctora year ago, who ordered a chest X-ray and chest CT; he hands you a CD with some cuts of the chest CT, which you review.
  • 30. 30
  • 31. 31 What approachto testing is most likely to yield a correct diagnosis? A. Repeat PFTs B. Serum antiprotease testing C. Bronchoscopywith biopsies D. Surgical lung biopsy E. Thorough occupational history We agree that B is the best answer. This patient has severe alpha-1 antitrypsin deficiency (A1ATD). Severe A1ATD is mistakenly believed to be rare; actually, 60,000 to 100,000 Americans are estimated to have the PI*ZZ genotype associated with severe deficiency (<50 mg/dL or 11 micromoles/L; >20 micromoles/L is normal). Most are of northern / western European descent and are undiagnosed. Features that should prompt consideration of A1ATD include:  Emphysema before age 45  Absence of smoking history or other exposures (e.g., organic dust)  Emphysema with basilar lucency  Unexplained liver disease  Family history of emphysema, liver disease, or panniculitis  Necrotizing panniculitis (rare, even in A1ATD patients)
  • 32. 32 The manifestations of A1ATD are complex and difficult to predict:  Many nonsmokers with PI*ZZ never develop liver or lung disease and seem to live a near-normal lifespan.  Smokers with severe A1ATD have wide variation in the rate of FEV1 decline and development of emphysema.  Many patients with A1ATD have an asthma-like phenotype with bronchodilator-reversible obstruction; these patients seem to be at greater risk for FEV1 decline (even if nonsmokers), and ATS recommends treating them "aggressively" with inhaled steroids and bronchodilators.  Liver disease is more common in childhood, but as many as 30-40% of patients aged >50 with A1ATD may have cirrhosis or carcinoma of the liver. Overall, prognosis in individual patients can't be predicted (except for those with already severe disease who smoke), becauseof the heterogeneity of the disease and the lack of prospective longitudinal studies. Patients with PI*MZand PI*SZ seem to be at increased risk for COPDif they smoke, but not markedly so if they do not. Somewhat surprisingly, the ATS recommends testing virtually all Americans with COPDfor A1ATD. (Also, anyone with unexplained liver disease, necrotizing panniculitis, asymptomatic obstruction on PFTs, and all siblings of A1ATD patients.) PI*MM (in 95% of U.S. population) ensures levels >20 micromoles/L and normal function. Nonrandomized trials suggest that A1AT supplementation may slow FEV1 decline and improve survival in severe A1ATD. Patients with lower FEV1 (31- 65% predicted) seemed to have a greater benefit. ATS guidelines are vague and acknowledge the relatively weak evidence, but suggest A1AT augmentation is appropriate for anyone with severe deficiency (<11 micromoles/L), and those with A1ATD (<20 micromoles/L) and either of these criteria:  FEV1 30-65% predicted (postbronchodilator);  Rapid decline of lung function (FEV1 decline >120mL/yr), regardless of initial FEV1. From observational data, the risk of anaphylaxis may be ~1% per patient over ~6 years of A1AT infusions. Patients should probably be given an Epi-Pen and taught to use it.
  • 33. 33 This patient's CT scanshows panacinar emphysema, with bullous disease worse at the bases. Panacinar disease can merge with severe centrilobular emphysema to create a nonspecific CT scan. Question 14 of 82 (score?) A 33 year old woman returns to see you for her third visit for significant progressive dyspnea. She has no other medical conditions, and normal imaging except for an enlarged pulmonary artery on CT scan. Her echocardiogram suggested severe pulmonary hypertension (with normal LV function) on her last visit, so you referred her for right heart catheterization. The report is: Initial values After 25 ppm NO inhaled Central venous pressure 8 mm Hg 9 mm Hg Pulmonary artery pressure 70/39 (mean 47 mm Hg) 63/36 (mean 43 mm Hg) PAOP ("wedge") 12 mm Hg 13 mm Hg Cardiac index 2.8 L/min/m2 2.9 L/min/m2 She's also had a negative V/Q scan, HIV test, sleep study, echocardiographic bubble study, liver and hepatitis panel, high-resolution CT scan, urine drug screen, and autoimmune labs. She is at normal weight and did regular aerobic exercise before, but now cannot walk 100 feet without stopping for dyspnea. Her oxygen saturations are normal with exertion. Jugular venous pressure is 11 cm. She has pitting edema over both tibias. What's the worstadditional therapeutic recommendation for her?
  • 34. 34 A. Bosentan B. Sildenafil C. Epoprostenol D. Nifedipine E. Warfarin We agree that D is the bestanswer. Idiopathic pulmonary arterial hypertension is a diagnosis of exclusion, arrived at after an extensive workup as outlined in guidelines and described above. All patients must undergo right heart catheterization before initiating treatment for any form of pulmonary arterial hypertension. A small percentage of patients are "vasoreactive" to adenosine or nitric oxide: they have a decrease in mean pulmonary artery pressure of >= 10 and to below 40 mm Hg when these agents are administered during RHC. These patients have an excellent prognosis as long as they take a vasodilating calcium channel blocker, such as nifedipine. Treatment for non-vasoreactive IPAH (as this patient has) may include oral endothelin receptorantagonists (bosentan), oral phosphodiesteraseinhibitors (sildenafil), or IV/inhaled prostanoids (epoprostenol, treprostinil, iloprost). U.S. guidelines recommend the more costly and inconvenient prostanoids for patients at "high risk" (rapid progression; RV dysfunction or RA pressure > 20 mm Hg; WHO functional class IV; high BNP; <300 m 6-minute walk). The 1A recommendations of the European Society of Cardiology are as follows: Evidence level WHO class II WHO class III WHO class IV I - A Ambrisentan, bosentan, sildenafil Ambrisentan, bosentan, sitaxetan, sildenafil, eprostenolIV, iloprost inhaled Epoprostenol IV
  • 35. 35 Diuretics should be prescribed when volume overload is present. Based on observational data of improved survival in patients with IPAH, warfarin is generally indicated as well. In those with an indwelling catheter for IV therapy, warfarin is believed to be additionally helpful in preventing catheter thrombosis. Atrial septostomyand lung (or heart-lung) transplantation are options when PAH medical therapies fail. Question 15 of 82 (score?) A 19-year old man presents to the ED with the suddenonset of mild dyspnea and chest pain. A chest X-ray reveals a right-sided pneumothorax, with 4.2 cm between the chest wall and visceral pleural line. He is uncomfortable but not in distress. Oxygen saturation is 95% on ambient air; heart rate is 95. There is no midline shift on the chest film; pulsus paradoxus is normal. This is the first time this has happened to him. He has always been healthy. You administer 100% humidified oxygen. What's the bestcourse of action over the next several hours? A. Continue 100% oxygen and obtain another chest film. B. Perform thoracentesis, aspirating as much air as possible. C. Place a Heimlich valve in the anterior chest; discharge home. D. Admit; place a chest tube to -20 cm wall suction. E. Admit; consult thoracic surgery for pleurodesis. We feelthat B and C are the best answers. This is a primary spontaneous pneumothorax (occurring without a clear cause, in the absenceof lung disease). Although administering 100% oxygen would result in slow resorption of the air and lung re-expansion, this is recommended only for small pneumothoraces. (The size of a pneumothorax is notoriously difficult to estimate on a chest film, but <3 cm between the lung edge and the chest wall has been suggested as a cut-off.) The besttreatment for the patient would be aspiration of air with a standard thoracentesis kit. After removal of air, leave the catheter in place with a closed stopcockand monitor for 6 hours. If a repeat chest film shows success and he does not live in a remote area, he can go home (without the catheter). No follow-up is necessary, unless symptoms return.
  • 36. 36 Alternatively, a Heimlich (one-way) valve could be attached to the thoracentesis catheter and he could go home afterward, returning in a few days for repeat film followed by removal of the catheter and valve. Chest tube placement could be performed, but would arguably be invasive and unnecessary, unless thoracentesis is unsuccessful(persistent air aspiration after 4 liters removed, suggesting a bronchopleural communication / leak). About half of those with PSP will never develop another pneumothorax. Pleurodesis could be considered for recurrent pneumothoraces Question 17 of 82 (score?) You are consulted by the ED for 40-year-old man presenting with shortness of breath and dry cough. He supervises construction and has been on-site for excavations in Tennessee, Ohio, Arizona, and Alabama, all in the last 2 months. He was diagnosed with pneumonia at an urgent care center 2 weeks ago and treated with levofloxacin for 1 week, with no improvement. Then he was prescribed prednisone 20 mg daily for 1 week, with steady worsening of his cough and subjective fevers. His temperature is 101, pulse 105, respirations 20, blood pressure 125/85. Oxygen saturation is 94% on room air. White blood cell count is 13,000 with 25% eosinophils. A chest X-ray shows patchy multifocal pneumonia. You schedule him for bronchoscopy. What do you most strongly recommend the ED physician provide now? A. Methylprednisone 125 mg IV. B. Itraconazole. C. Piperacillin and vancomycin. D. Amphotericin B. E. Heparin. We feelthat B is the best answer. In this patient with a pneumonia that has been unresponsive to broad-spectrum antibiotics and a courseof corticosteroids, with travel & exposure history suspicious for fungal infection, empiric antifungal therapy should be provided.
  • 37. 37 Coccidioides is the only dimorphic fungus that causes peripheral eosinophilia. Itraconazole or fluconazole are the preferred treatments for non-life-threatening coccidioidomycosis, with amphotericin B reserved for severe or disseminated infections. These agents are also effective treatment for histoplasmosis and blastomycosis (with ampho-B reserved for severe infections). There is insufficient data to supportthe use of voriconazole or posaconazole. Diagnosis of coccidiodomycosis is strongly suggested by positive serology, because most people lose seropositivity to coccidioides within 3 months of a resolved infection. (Positive serologies are also useful in acute histoplasmosis, but not in blastomycosis.)Negative serology does not rule out any fungal infection, because serology is insensitive in early infection. A new urine antigen test to coccidioides is available and should be checked if serologies are negative. Urine antigen tests are also available for histoplasmosis and blastomycosis (there is cross-reactivity making these assays only ~80% specific, but it's of no clinical consequencesince the treatments for both are the same). Histopathology / biopsycan demonstrate fungal elements using proper staining (e.g., spherules in coccidioidomycosis), orgranulomas in histoplasmosis. Culture from BAL or biopsy are high-yield but can take weeks to grow. Apparently, bronchoalveolar lavage is safe in coccidioidomycosis (although I'd wear an N95 mask and do it in a negative pressure room in suspected cases anyway), but handling cultures is dangerous and requires special measures; the lab should be notified if coccidioides is suspected. Regarding the other answer choices:eosinophilic pneumonia should be considered, but the lack of responseto an initial courseof steroids (albeit low dose)and the risk factors for fungal infection would make high dosesteroids ill-advised. Pulmonary embolism would not likely present with multifocal opacities. Question 18 of 82 (score?) Friends of a 21-year-old woman call 911 becauseshe is found unconscious alone in a room at a party. When EMS arrived, the woman had a GCS scoreof 8, pulse of 65, and respiratory rate of 12 / min, and was intubated in the field. She was brought to the emergency department, where you see her an hour later. She is now lethargic but arousable, moves all extremities and opens her eyes to voice. Pulse is 78, and respirations 18 / min. Physical examination, ECG, and chest film are all normal.
  • 38. 38 Friends say there was alcohol but no drug use other than marijuana at the party. They say their friend is a full-time college student who only drinks when at parties. Ethanol level, chemistry panel and toxicology screen, and other labs are all pending. What is the bestnext step? A. Give flumazenil B. Give naloxone C. Place a nasogastric tube and give activated charcoal D. Observe, with a plan to extubate and discharge from the ED within a few hours E. Give calcium gluconate We feelthat D is the best answer. This young woman is likely intoxicated with ethanol. She seems to be recovering steadily. In the absence of evidence of other intoxications or poisoning, the fewer aggressive interventions she undergoes, the better. It could be argued that since she had a normal respiratory rate and other vital signs, with no evidence of trauma, she didn't need to be intubated at all. The safety of forgoing intubation in severely alcohol-intoxicated patients with GCS < 6 was demonstrated in a sample of 405 drunken revelers needing medical attention during Oktoberfestin Munich in 2004. Other intoxications or poisonings should be considered, but should only be treated when there is suspicion of a specific ingestion/intoxication. The emergency department has done that by ordering appropriate labs and tests and by interviewing her friends. Naloxone should be given where there is known or suspected opioid toxicity. Flumazenil should not be administered, even in cases of benzodiazepine overdose, mainly becauseof its potential for causing withdrawal seizures. Thiamine should be given in chronic alcohol abusers, but not necessarily to all intoxicated patients. Activated charcoal is useful in gastric decontamination after ingestion of high- molecular weight compounds (e.g., acetaminophen or other oral medications), but should not be administered unless an ingestion is suspected. Charcoal's efficacy rapidly falls after the first hour after ingestion; evidence for its benefits is low, overall. Hemodialysis can rapidly remove small molecular weight compounds, such as methanol and ethylene glycol (and their toxic metabolites) and lithium.
  • 39. 39 (Table: Common Toxidromes Signs & Symptoms, from fmpe.org) Question 19 of 82 (score?) Sally K. has COPD with an FEV1 of 48% predicted. You referred her to a lung rehabilitation program last year, which she completed and which made her feel better. When her insurer stopped paying for the visits, though, she stopped the program and returned to a sedentary lifestyle. She comes to see you today and wants to know what kind of exercise program would be bestfor her now. She insists on starting only one form of exercise. What exercise program do you recommend? A. Aerobic exercise using the legs (e.g., walking) B. Unsupported arm exercise, low weight, high repetitions C. Unsupported arm exercise, high weight, low repetitions D. Inspiratory muscle training E. Recommend holding off until a cardiac stress test can be obtained We feelthat A is the best answer. Aerobic exercise should be the cornerstone of any exercise program for someone with COPD. Increased cardiopulmonary fitness due to aerobic exercise is believed to be the major source of benefit from pulmonary rehabilitation, as this patient has experienced herself. Aerobic exercise has multiple other beneficial effects on mood, reduced cardiovascular risk, improved self-image and quality of life. These benefits have been demonstrated in over 20 randomized controlled trials (although, it should be noted, benefits were inconsistent & heterogeneous across trials). One study also shows improvement in muscle mass associated with aerobic exercise in pulmonary rehab. The aerobic exercise component of pulmonary rehabilitation should ideally be lifelong; its short-term nature is a function of the limits set by the payer system. (In 2011, Medicare paid outpatient centers $28 per rehabilitation session, per patient -- which may help explain the low availability of pulmonary rehab programs in many areas.)
  • 40. 40 Limited studies show that people with COPD are quite inactive. There is no proven effective method of motivating people to exercise. Some experts recommend tying exercise to social interaction by exercising with a friend. Varying the routine might help. Simple brisk walking is all that's needed to get the benefits of exercise, but if this patient wants to try something "new," there are endless variations on aerobic exercise, with programs offered through fitness centers, on TV, and online. Many patients believe that dyspnea is in itself dangerous, and need reassurance that mild sustained dyspnea during exercise is safe (& in fact is the goal). Inspiratory muscle training has shown benefit in some trials but should be considered adjunctive. By no means should it replace aerobic exercise. Strength training is also beneficial, and can be added to an aerobic exercise program in highly motivated patients. Question 20 of 82 (score?) A 65-year-old man presents with symptoms of myasthenia gravis and a chest CT is performed, showing a large anterior mediastinal mass. A needle biopsyconfirms thymoma. The mass is locally advanced, compressing the superior vena cava (SVC) but not clearly invading the SVC or the pericardium. He has mild COPD with FEV 70% predicted with no cardiac history. Which of the following statements is true? A. An attempt at surgical resection should be made. B. The mass is unresectable; refer for radiation. C. Myasthenia symptoms suggest a poorprognosis. D. Neoadjuvant chemotherapy could improve resection and survival. E. Absence of malignant features on histology is the strongest predictor of survival. You answeredD. We agree that A and D are the best answers. Thymomas and thymic carcinomas are neoplasms which may arise from the thymus. In any case where there is not clear invasion into mediastinal structures
  • 41. 41 (blood vessels, pericardium, pleura), surgical resection should be attempted. Retrospective series strongly suggest surgical resection offers the bestcure, although there are no randomized controlled trials of surgery vs. nonsurgical treatments. This patient's mild lung disease should not preclude surgery. The histopathology of thymic neoplasms is complex and has an uncertain contribution to prognosis, because histology is heterogeneous within the tumor (e.g., carcinoma and thymoma may exist simultaneously in different areas). Invasion through the thymic capsule into surrounding tissues does diminish expected survival. However, even with a small degree of local invasion, 5-year survival with resection is >80%. There are multiple staging systems, with the Masaoka system most widely used. Myasthenia gravis (MG) symptoms are present in up to 50% of people with thymomas, but are rare in thymic carcinoma. Symptoms of MG are associated with less advanced disease, and resection of the thymoma reduces myasthenia symptoms (without eliminating them) in most patients. Palliative radiation is appropriate for cases of unresectable disease. Adjuvant radiation therapy after resection is strongly recommended after incomplete resection. After cases of complete resection, adjuvant radiation has an unlikely benefit except perhaps in stage III disease. Thymomas are chemotherapy-sensitive. Neoadjuvant chemotherapy may be used for large bulky masses that on imaging, appear too technically difficult to resect completely. There are multiple chemotherapy regimens, most of which are based on cisplatin (e.g., adding doxorubicin, cyclophosphamide and prednisone). Followup should continue for at least 10 years. Chemotherapy is the preferred treatment for recurrent or metastatic disease. Question 21 of 82 (score?) A 40 year old man underwent bilateral lung transplantation 8 months ago for cystic fibrosis. He is both a CMV and EBV mismatch (donorseropositive, recipient
  • 42. 42 seronegative). He has done relatively well, but today presents with weight loss, fever, and night sweats for 2 weeks. A chest CT shows more than 10 lung nodules bilaterally, varying in size between 0.5 and 4.0 cm, and and mild mediastinal lymphadenopathy. He is taking tacrolimus, mycophenolate mofetil, and prednisone. He undergoes CT-guided biopsy of a large peripheral nodule, and it demonstrates innumerable clustered lymphocytes. Flow cytometry establishes a polyclonal B- cell population. Culture of the biopsy is pending. An interferon-gamma release assay is also pending. What's the best next step? A. Initiate anti-bacterial and anti-tuberculosis therapy, pending culture results. B. Initiate ganciclovir therapy. C. Reduceimmunosuppression. D. Perform bronchoscopywith EBUS-guided biopsy of mediastinal lymph nodes. E. Consult thoracic surgery for mediastinoscopy. We feelthat C is the best answer. Post-transplant lymphoproliferative disorder (PTLD) results from EBV infection of lymphocytes (of donoror recipient origin), inducing them to proliferate and aggregate where they shouldn't. More than 80% of cases involve B-cell proliferation; 15% are from T-cells. PTLD occurs in 2-9% of lung transplant recipients, mostly among those who were EBV-seronegative before transplant. Multiple well-formed lung nodules is the usual presentation, often with constitutional symptoms. PTLD tends to present at an advanced stage in lung transplant recipients. It may involve other organs (including the central nervous system, abdominal organs, and skin). Pathology demonstrates polyclonal proliferation of lymphocytes. (Non-Hodghkin lymphoma can also result, which would result in a monoclonal lymphocyte population on biopsy.) Reduction in immunosuppression permits the host immune system to suppress EBV replication and can result in regression of lung lesions and symptoms in PTLD. However, this increases the risk of acute rejection. Other treatments have been reported to be successfulin case reports/series:  Surgical excision  Radiation therapy
  • 43. 43  Chemotherapy  Extracorporeal photochemotherapy  Retransplantation  Ganciclovir, rituximab, interferon alfa Overall, the prognosis of PTLD is guarded, due to the risks of recurrence, and rejection with reduction in immunosuppression. Question 22of 82 (score?) This 48 year old man had right middle and lower lobectomies last year for non- small cell lung cancer. He did relatively well since, but has now been admitted from oncology clinic with a fever and a high white blood cell count, and BP of 95/60 with a HR of 125. He is not immune-suppressed and last saw a doctor4 months ago. Doses of levofloxacin and azithromycin have been given.
  • 44. 44
  • 45. 45
  • 46. 46 As the best next step, you recommend: A. Thoracentesis B. Bronchoscopywith bronchoalveolar lavage C. Tube thoracostomy D. Expedited radiation and salvage chemotherapy E. Change antibiotics to piperacillin/tazobactam, vancomycin and amikacin. We feelthat C and A are the best answers. This may be an empyema, and should be considered one until proven otherwise. The contrast-enhancing, thickened visceral and parietal pleura on the CT images are a good example of the "split pleura sign," suggestive of empyema or
  • 47. 47 hemothorax. (A thickened pleura can also be present after lobectomy alone, so this sign is less specific here.) Thoracentesis would not be inappropriate, but we felt definitive drainage was best given the patient's sepsis. Note that the large apparent effusion on scoutchest CT is nearly all due to the opacity created by an elevated liver/diaphragm due to volume loss after his large lung resection (see the liver at the level of the heart, on axial cuts). Ultrasound would help avoid sticking this with a thoracentesis needle. The actual empyema volume is small. (Cultures of the pleural fluid grew S. pneumoniae). This patient got a chest tube; these images were actually post-chesttube removal (the original CT showed an identical-sized fluid collection), which is likely responsible for the gas bubbles in the effusion. Without recent health-care contacts or immune suppression, empiric antibiotics for community-acquired pneumonia are appropriate, and bronchoscopywould not be expected to help. Even if the effusion were malignant, drainage would be indicated before further cancer treatments. Question 22 of 82 (score?) You've just diagnosed a 32 year old woman with idiopathic pulmonary arterial hypertension (IPAH) based on the right heart catheterization results below. She had been lost to followup after her initial evaluation months ago; she now has dyspnea at rest, lower extremity and sacral edema, and is housebound and requiring assistance with housekeeping. SaO2 is 85% during ambulation. Right atrial pressure 14 mm Hg Right ventricle pressure 69/16 mm Hg Pulmonary artery pressure 69/30 mm Hg Mean PA pressure 46 mm Hg
  • 48. 48 Pulmonary capillary wedge pressure 11 mm Hg Cardiac index 1.6 L/min/m2 Pulmonary vascular resistance 11 Wood U Infusion of nitric oxide reduces the mean PA pressure to 39 mm Hg. The cardiac index also falls, to 1.3 L/min/m2. You prescribe warfarin, oxygen therapy, and furosemide. The best choice for additional therapy is: A. Oral sildenafil. B. Oral bosentan. C. Inhaled treprostinil. D. I.V. epoprostenol. We agree that D is the bestanswer. This patient is WHO Class IV, the most advanced stage of PAH, with dyspnea at rest and evidence of right heart failure on exam (anasarca). U.S. and European expert guidelines recommend intravenous prostacyclin analogues (e.g., epoprostenol, treprostenil) for WHO Class IV PAH, as there is a larger bodyof evidence for efficacy with these agents (particularly epoprostenol)than for oral or inhaled agents. Vasodilator agents have not been compared well head-to-head, and for less-severe PAH (WHO class III and below) various agents (oral, IV, or inhaled) are considered appropriate, depending on the patient's particular severity of disease, personal preferences, and other risk factors that may be present. The European Society of Cardiology's 1A recommendations (strong, based on high-quality evidence) for PAH are:
  • 49. 49 Evidence level WHO class II WHO class III WHO class IV I - A Ambrisentan, bosentan, sildenafil Ambrisentan, bosentan, sitaxetan, sildenafil, epoprostenolIV, iloprost inhaled Epoprostenol IV Warfarin, oxygen, and diuretic therapy should be standard treatment for advanced IPAH, barring contraindications. A vasodilator responseis indicated by a drop in mean PA pressure by 10 mm Hg to less than 40 mm Hg, without a drop in cardiac output. Question 23 of 82 (score?) You are asked your opinion about a 52 year old woman with pleural effusions. She has decompensated congestive heart failure, but because her effusions were asymmetric, the admitting physician performed thoracentesis. 1,500 mL of straw- colored fluid were removed from the left hemithorax, with relief of dyspnea. Since admission, the patient has received diuretics with further reduction in dyspnea. Your colleague would like help interpreting the results of the pleural fluid studies:  Glucose 85 (normal)  pH 7.38 (normal)  Protein 4.0 g/dL (serum: 7.4; ratio = 0.56)  LDH 230 (serum: 290; ratio = 0.79)  N-terminal brain natriuretic peptide of pleural fluid 1,800 pg/mL Based on the information you have, what's the best way to describe this effusion? A. It's exudative, because the findings meet Light's criteria. B. It's transudative, because the pH and glucose are normal. C. Tube thoracostomywill likely be necessary for complete drainage. D. It's due to congestive heart failure, as demonstrated by the clinical picture and markedly elevated NT-BNP. E. It's not possible to say, from the information provided.
  • 50. 50 We feelthat D is the best answer. Light's criteria* are sensitive, but not specific for exudative pleural effusions. About 25% of transudative effusions may be misclassified as exudates by the criteria. The transudative effusions of congestive heart failure patients, drained after diuretic therapy is given, are commonly misclassified as exudates this way Presumably, the diuretics remove water and increase the concentration of protein and LDH in pleural fluid. A gradient of more than 3.1 g/dL in protein concentration between the pleural fluid and serum (as seen here) correctly reclassifies many effusions as transudates. In effusions due to CHF, N-terminal BNP in the pleural fluid > 1,500 pg/mL is better than the serum-pleural protein gradient at accurately reclassifying "exudates" as transudates. Notably, pleural fluid BNP is much less reliable than pleural fluid NT-BNP for this purpose. Properly reclassifying a CHF-related effusion as a transudate spares the patient unnecessary further testing and treatment. Therapeutic thoracentesis could be provided to relieve severe dyspnea, but tube thoracostomygenerally should not be performed. * Light'scriteria: Pleural fluid-to-serum protein ratio > 0.5; LDH in pleuralfluid > 60% of serum value; LDH > 2/3 the upper limit of normalfor serum. Question 24 of 82 (score?) A victim of a house fire is brought to your ICU from the ED. He was found in his garage where he has a sophisticated workshop, according to a neighbor, but no one knows what chemicals may have been present. He was comatoseand was intubated in the field. He is only minimally burned; a rapid bronchoscopyin the ED showed no serious mucosal injury. His vital signs: HR 110, BP 100/70, easily bag-mask ventilated, oxygen saturation of 94% with an FiO2 of 0.90. His ABG is 7.20 / 44 / 80, with 24% methemoglobin. Lactic acid is 7.5. The ED MD placed a central venous catheter, and central venous oxygen saturation is 90%, with a pO2 in venous blood of 68.
  • 51. 51 What therapy do you prioritize first? A. Hydroxycobalamin and sodium thiosulfate B. Amyl nitrite C. Pralidoxime D. Hyperbaric oxygen E. Benzodiazepines You answeredA. We agree that A is the bestanswer. Up to 35% of fire victims have cyanide poisoning. Incinerated plastics, rubber, polyurethane, and melamine (in household goods)can all release lethal inhaled doses ofcyanide. Sodium nitroprusside in prolonged infusions or in people with renal failure is another cause. Cyanide disrupts mitochondrial oxidative phosphorylation and aerobic metabolism of glucose. Signs of cyanide poisoning include a high venous pvO2 with a low venous-arterialpO2 gradient (due to inadequate peripheral tissue oxygen use) and an anion-gap lactic acidosis. Symptoms can affect virtually all organ systems, the most serious including coma; pulmonary edema with respiratory failure; and hypertension, followed by hypotension, bradycardia, ventricular dysrrhythmias and cardiopulmonary collapse. Cyanide poisoning is rare but lethal, and definitive diagnosis (by serum cyanide level) may require hours. Therefore, empiric therapy should be given rapidly if cyanide toxicity is suspected. Sodiumthiosulfate is a sulfur donor that transforms cyanide to a less-toxic compound, and should be given to all suspected victims. Amyl nitrite is a standard co-antidote, working by producing methemoglobin, which then binds and detoxifies cyanide. (Sodium nitrite or dimethylaminophenol also induce methemoglobinemia and are alternate agents; methylene blue releases free cyanide and should not be given if cyanide toxicity is suspected). This patient already has serious methemoglobinemia likely due to carbonmonoxide inhalation, however, and worsening this could be catastrophic. Hydroxycobalamin directly binds and detoxifies cyanide, without producing methemoglobin, and should be administered with sodium thiosulfate. Hyperbaric oxygen may reduce cognitive sequelae of carbon monoxide poisoning (although a Cochrane review casts doubt on this), but shouldn't take first priority.
  • 52. 52 Pulse oximetry and arterial blood gas can be unreliable in cases of severe smoke inhalation. Carbon monoxide poisoning increases carboxyhemoglobin, falsely raising SaO2; methemoglobinemia tends to push SaO2 toward 85% (due to equal absorption of oxyhemoglobin and deoxyhemoglobin wavelengths). Methemoglobinemia can also cause ABG to overestimate SaO2, but is helpful in showing the levels of carboxyhemoglobin and methemoglobin. Co-oximetry measures 4 wavelengths of light (compared to regular oximetry's 2) and is necessary to accurately identfy the oxyhemoglobin level (saturation) in cases of carbonmonoxide poisoning. Cyanide affects virtually all bodytissues, attaching itself to ubiquitous metalloenzymes and rendering them inactive. Its principal toxicity results from inactivation of cytochrome oxidase (at cytochrome a3), thus uncoupling mitochondrial oxidative phosphorylation and inhibiting cellular respiration, even in the presence of adequate oxygen stores. Cellular metabolism shifts from aerobic to anaerobic, with the consequent production of lactic acid. Consequently, the tissues with the highest oxygen requirements (brain and heart) are the most profoundly affected by acute cyanide poisoning. Smoke inhalation, suicidal ingestion, and industrial exposures are the most frequent sources of cyanide poisoning. Smoke inhalation Studies in France, Sweden, and Scotland, as well as in the United States, have documented smoke inhalation as an important sourceof cyanide poisoning. Individuals with smoke inhalation from enclosed spacefires who have sootin the mouth or nose, altered mental status, or hypotension may have significant cyanide poisoning (blood cyanide concentrations >40 mmol/L or approximately 1 mg/L). Many compounds containing nitrogen and carbon may producehydrogen cyanide (HCN) gas when burned. Some natural compounds (eg, wool, silk) produceHCN as a combustion product. Household plastics (eg, melamine in dishware, acrylonitrile in plastic cups), polyurethane foam in furniture cushions, and many other synthetic compounds
  • 53. 53 may producelethal concentrations of cyanide when burned under appropriate conditions of oxygen concentration and temperature. Intentional poisoning Cyanide ingestion is an uncommon, but efficacious, means of suicide, often involving cyanide salts found in hospital and research laboratories. Not surprisingly, individuals in certain occupations, suchas health-care and laboratory workers, are at risk for suicidal ingestion of cyanides. Industrial exposure Countless industrial sources ofcyanides exist. Cyanides serve an extremely important role in the metal plating and recovery industries. In addition, industry uses cyanides in the manufacture of plastics, as reactive intermediates in chemical synthesis, and as solvents (in the form of nitriles). Exposure to salts and cyanogens occasionally causes poisonings; however, a significant risk for multiple casualties occurs when these products comeinto contact with mineral acids because HCN gas is produced. Water contactwith the soluble salts (eg, potassium, sodium cyanide) also may liberate HCN. Iatrogenic exposure Sodium nitroprusside, when used in high doses orover a period of days, can producetoxic blood concentrations of cyanide. Patients with chronic renal failure, pediatric patients, and those with low thiosulfate reserves (eg, malnourished, postoperative) are at increased risk for developing symptoms, even with therapeutic dosing. Resultant confusion and combativeness initially may be mistaken as intensive care unit (ICU) syndrome (ie, sundowning). Problems may be avoided by coadministration of hydroxocobalamin or sodium thiosulfate. Ingestionof cyanide-containing supplements Ingestion of cyanide-containing supplements is rare. Amygdalin (synthetic laetrile, also marketed as vitamin B-17) contains cyanide and can be found in the pits of many fruits, such as apricots and papayas; in raw nuts; and in other plants (lima beans, clover, and sorghum). The substancewas thought to have anticancer properties due to the action of cyanide on cancer cells. However, laetrile showed no anticancer activity in human
  • 54. 54 clinical trials in the 1980s and is not available in the United States,[2] although it can be purchased on the Internet. EmergencyDepartment Care Initial emergency department care for patients with cyanide exposure is identical to that provided in the prehospital phase. Provide supportive care, including the following:  Airway control, ventilation, 100% oxygen delivery  Crystalloids and vasopressors, as needed, for hypotension  Sodium bicarbonate titrated according to arterial blood gas (ABG) and serum bicarbonate level Decontaminate the patient with removal of clothing/skin flushing and/or activated charcoal (1g/kg), as appropriate. Activated charcoal should be given after oral exposure in alert patients who are able to protect the airway or after endotracheal intubation in unconscious patients. Remember to protectthe health-care provider from potential contamination. Administer cyanide antidotes if the diagnosis is strongly suspected, without waiting for laboratory confirmation. The antidotes include hydroxocobalamin (Cyanokit) and the Cyanide Antidote Kit, which includes amyl nitrite pearls, sodium nitrite, and sodium thiosulfate. Cyanokit Hydroxocobalamin (Cyanokit), routinely used in Europe, has been approved by the US Food and Drug Administration (FDA) for treating known or suspected cyanide poisoning.[7, 8] Hydroxocobalamin combines with cyanide to form cyanocobalamin (vitamin B- 12), which is renally cleared. Hydroxocobalamin administration resulted in faster
  • 55. 55 improvement in mean arterial pressure but similar mortality and serum acidosis, as compared with sodium nitrite, in animals.[9] A repeat doseof hydroxocobalamin and/or coadministration of sodium thiosulfate (through a separate line or sequentially) have been suggested to improve detoxification and are recommended in patients with continuing elevated lactate levels or continuing signs of cyanide toxicity.[10, 11] Adverse effects of hydroxocobalamin administration include transient hypertension (a benefit in hypotensive patients), reddish brown skin, mucous membrane and urine discoloration, and rare anaphylaxis and anaphylactoid reactions. It also interferes with co-oximetry (about a 5% increase in carboxyhemoglobin levels) and blood chemistry testing (bilirubin, creatinine kinase and possibly liver enzymes, creatinine, phosphorus,glucose, magnesium, and iron levels) due to its bright red color.[12] It can also interfere with hemodialysis.[13] Certain medications should not be administered simultaneously or through the same line as hydroxocobalamin, including diazepam, dopamine, dobutamine, and sodium thiosulfate. Cyanide Antidote Kit The Cyanide Antidote Kit contains amyl nitrite pearls, sodium nitrite, and sodium thiosulfate. Amyl and sodium nitrites induce 15-20% methemoglobinemia in red blood cells, with methemoglobin combining with cyanide and releasing cytochrome oxidase enzyme. Inhaling crushed amyl nitrite pearls is a temporizing measure before IV administration of sodium nitrite. Sodium thiosulfate enhances the conversion of cyanide to thiocyanate, which is renally excreted. Thiosulfate has a somewhat delayed effect and thus is typically used with sodium nitrite for faster antidote action. Avoid the nitrite portion of the kit in patients with smoke inhalation unless carboxyhemoglobin concentration is very low (< 10%). The induction of methemoglobinemia from the nitrites, in addition to present carboxyhemoglobinemia, significantly reduces the oxygen-carrying capacity of blood. In patients with preexisting anemia, the sodium nitrite doseneeds to be reduced dosein proportion to the hemoglobin concentration. Consult a regional toxicology center for appropriate dosing.
  • 56. 56 Vasodilatation leading to hypotension is another adverse effect of the Cyanide Antidote Kit. Question 25: A 51 year old woman is referred to you by her infectious disease physician for a second opinion. She has had cough and worsening dyspnea for a few months. For this, she underwent open lung biopsy elsewhere; transbronchial biopsies showed noncaseating granulomas. She was diagnosed with sarcoidosis, and her ID doctor wants your OK before starting steroids. She has HIV, well-controlled on antiretroviral therapy with a CD4 count of 550. Exam is notable for oxygen saturation of 92% on ambient air and faint crackles at the lung bases. PFTs showa mild restrictive defect and moderate diffusion impairment. Chest X-ray is within normal limits and shows no hilar lymphadenopathy. Chest CT images are below.
  • 57. 57
  • 58. 58 You review the pathology results, which describe"noncaseating granulomas and monotonous sheets of T and B lymphocytes in the lung interstitium." What's the most likely diagnosis? A. Lymphocytic interstitial pneumonia. B. Primary pulmonary lymphoma. C. Secondarypulmonary lymphoma. D. Sarcoidosis. E. Pneumocystis pneumonia. We feelthat A is the best answer. Lymphocytic interstitial pneumonia is an uncommon condition that usually occurs in association with some form of immune dysfunction or dysproteinemia, such as autoimmune disease, HIV or EBV infection, or multiple myeloma. A few cases may occurwithout such comorbidities. In LIP, sheets of polyclonal T and B lymphocytes infiltrate the lung interstitium, causing a variable degree of dysfunction. Clinical:Cough and dyspnea are most common, but systemic/constitutional symptoms can occur. Hypoxemia and clubbing are common in more severe disease.
  • 59. 59 Diagnosis:Requires open lung biopsyfor confirmation. Besides sheets of polyclonal lymphocytes, plasma cells and macrophages, giant cells, noncaseating granulomas, amyloid deposition, lymphoid germinal centers, and fibrosis may all be found on biopsy. Dominant findings are in the interstitium but may "spill" into the airspaces. Imaging:Chest film with reticular opacities later coalescing into airspace/interstitial pattern. Chest CT with centrilobular nodules; ground glass; thickened interlobular septa; later with cystic airspace disease and consolidation. Late honeycombing is possible. PFTs:Usually restrictive, with a diffusion impairment. Bronchoscopy: Lavage shows an abundant T-cell lymphocytosis. Treatment:Corticosteroids have been reported to be beneficial in case series. Prognosis: Unpredictable, varying from indolent to progressive and fatal. Some patients' pulmonary disease may progress despite corticosteroids. Since LIP is usually a feature of an immune condition, prognosis is believed to be dependent on the underlying condition. Although LIP may progress to a low-grade pulmonary or systemic lymphoma, this seems to be uncommon (~5%). Sarcoidosis is unlikely in the absence of lymphadenopathy, and does not have sheets of lymphocytic infiltration on biopsy. Lymphomas are generally defined as monoclonal, not polyclonal B or T cell populations. Pneumocystis pneumonia is unlikely in well controlled HIV at this CD4 count.
  • 60. 60 Question 26of 82 (score?) A patient taking pramipexole for restless legs syndrome for the past 2 years complains of symptoms worsening and occurring earlier in the day, and feeling symptoms in her arms as well as legs. Ferritin is 125 (normal). What do you recommend? A. Add another doseof pramipexole earlier in the day. B. Switch to a benzodiazepine 1-2 hours before bed. C. Switch to levodopa 1-2 hours before bed. D. Start iron replacement therapy. You answeredA. We agree that A and B are the best answers. Augmentation of restless legs syndrome (increase in symptom severity, occurrence earlier in the day, spreading to other bodyparts) is less common with dopamine agonists than with levodopa, but can occurand is more common after years of therapy. Interestingly, increasing dopamine-agonist therapy by adding a doseearlier in the day can diminish augmentation symptoms, and would be reasonable to try. Switching to another dopamine-agonist (e.g. ropinirole) would be reasonable. Levodopais a viable option but is recommended by an expert advisory panel to be used only for intermittent RLS. Benzodiazepines at night (diazepam, clonazepam) are a well-established therapy for RLS and could be substituted for the pramipexole. Carbergoline works well for RLS but is used rarely (it causes cardiac valve disease when used at high doses for Parkinson's). Gabapentin, opioids (codeine, methadone, tramadol, etc), pregabalin, and other drugs have been shown beneficial for RLS in studies of varying quality.
  • 61. 61 Iron deficiency should be treated if present in people with RLS, but a normal ferritin rules it out in the absence of acute illness. Iron deficiency itself is not known to cause augmentation. Question 27 of 82 (score?) It's 1962, and you work for an insurance company. It's a boring job but they assign you an interesting project. They're pretty sure smoking causes lung cancer, and they want to prove it so they can defend raising premiums on all their smokers. What's the most appropriate study design to answer the question of whether smoking causes lung cancer? A. Randomized controlled trial. B. Case series review. C. Cohortstudy. D. Case-controlstudy. E. Telephone survey. We feelthat C is the best answer. Randomized controlled trials have the greatest strength in proving causation. However, randomizing people to smoke or not to smoke would be unethical. A cohort study is the next best study design to prove causality. In a prospective cohortstudy, one cohortof subjects with the exposure of interest (i.e., smokers) and one without (non-smokers) are followed over time for the development of the outcome of interest (lung cancer). With large enough cohortsizes, other potentially confounding factors can be controlled for, and the independent contribution of the exposure determined with a high degree of confidence. The main downside of cohortstudies is that they take a long time (years). In a case-controlstudy, cases (i.e., people with lung cancer) are identified within a database, and are compared against controls (people without lung cancer) across multiple factors. Statistical analysis identifies factors (e.g., smoking, age, diet) that are distributed differently between cases and controls. A factor that is much more prevalent in lung cancer cases (such as smoking) is then identified as a risk factor. Case-controlstudies can be donerapidly and cheaply compared to randomized
  • 62. 62 trials and cohortstudies, but can only establish associations between exposures and outcomes;case-controlstudies can't prove causation. Case series and telephone surveys aren't controlled, and can't establish causation at all. At most, they might generate a hypothesis for a well-conducted clinical study of one of the above types. Question 29 of 82 (score?) A 58 year old man in your clinic has chest pain and palpitations. He becomes diaphoretic; blood pressure is 80/60. According to ACLS algorithms, what is the next thing you should do? A. Perform synchronized electrical cardioversion. B. Give a chewed 160-325 mg aspirin. C. Obtain IV access. D. Give amiodarone 300 mg IV. E. Perform unsynchronized cardioversion (defibrillation dose). You answeredA. We agree that A is the bestanswer. This is symptomatic or unstable ventricular tachycardia, based on hypotension (systolic BP < 90) and symptoms (chest pain). Current ACLS guidelines (good through 2015) advise immediate electrical cardioversion. Shocks of 50-100 joules are appropriate initially, escalating to 200 J if unsuccessful.
  • 63. 63 Question 30of 82 (score?) You are consulted for pleural effusions in a 61 year old man directly admitted to the hospital an hour ago. He presented with progressive dyspnea and leg swelling and no other symptoms. He has jugular venous distension, dullness at the lung bases, and pitting leg edema. Vitals are all normal except for BP 145/90. He has NYHA III symptoms (dyspneic walking across the room). He has received no treatments yet. Chest X-ray shows moderate-sized bilateral pleural effusions and an enlarged cardiac silhouette. An echocardiogram and labs are pending. What's the best initial management for this patient's pleural effusions? A. Diagnostic thoracentesis (unilateral). B. Diagnostic thoracentesis (bilateral). C. Therapeutic thoracentesis (bilateral, with a chest film in between). D. Diuretics and afterload reduction. E. All of the above. We feelthat D is the best answer. Clinically, this patient has congestive heart failure. It's overwhelmingly likely that the effusions are due to his CHF, and a diagnostic thoracentesis is not necessary. A few proposedindications for performing diagnostic thoracentesis in people with CHF-related pleural effusions are:  Fever (to diagnose a possible pleural infection)  Grossly asymmetrical effusions  Chest pain (suggesting an inflammatory process affecting the pleura) None of these are present here. Therapeutic thoracentesis is reasonable in people with severe dyspnea due to decompensated CHF, but not as initial therapy before first-line, noninvasive, medical therapy has been provided (diuretics and afterload reduction with vasodilator therapy). These may rapidly eliminate the effusions and symptoms.
  • 64. 64 Question 32 of 82 (score?) Mr. R, your 76-year old man in bed 3, just experienced a cardiac arrest on ICU day 6, after admission for septic shocklast week. His lack of other medical problems, along with a reduced need for his norepinephrine infusion and mechanical ventilatory support, had led you to hope he was improving. This morning, though, his blood pressure abruptly plummeted and he had a PEA arrest, with recovery of spontaneous circulation after 9 minutes of ACLS. After the code, you collect your thoughts and head out of the room to talk to his wife. After hearing you explain the situation, she asks, "Doctor, what are my husband's chances of coming home from the hospital to live a normal life with me?" You gently inform her it will require a few more days to provide the most accurate prognosis. At this point in time (immediately post-resuscitation in the ICU), based on large observational series, what are Mr R's chances for being discharged home with a good neurologic outcome? A. 48% B. 32% C. 16% D. 4% E. It's impossible to say. You answeredD. We agree that D is the bestanswer. Prognosis after a first cardiac arrest occurring in the ICU is poor. An observational study of almost 50,000 such arrests showed an overall survival to hospital discharge of 16%. Requiring vasopressors priorto the arrest was a major discriminator in outcome:
  • 65. 65  Only 10% of patients on pressors prior to arrest survived to discharge, and only 4% were discharged home (the others went to rehab or long-term acute care).  Among those with PEA/asystole despite pressors,only 1.7% were able to perform their own activities of daily living at the time of discharge.  People with ventricular fibrillation or tachycardia not requiring pressors prior to arrest did much better: 40% survived, 20% went home, and 17% had good neuro outcomes.  These data did not provide longitudinal outcomes data post-hospital discharge (e.g., neurological & functional recovery after 6 months); however, those outcomes are usually also poor(see below). Physicians do not usually convey prognosis after cardiac arrest (or critical illness in general) accurately or effectively to patients' families (according to those families' responseto surveys). Although such discussions are difficult for all parties involved, families require and deserve this information as they struggle with their roles as surrogate decision makers. Although some people discharged to rehabs or LTACs may recover function, it would be dishonest to give false hope by suggesting this as anything other than a long shot, especially after an in-ICU cardiac arrest. Only a minority of people survive one year after LTAC admission, and many fewer recover to live without needing major assistance. For example, in a 2009 study of 126 patients receiving tracheostomies for prolonged mechanical ventilation for various causes of respiratory failure, 93% of surrogates expected their loved one to survive one year, recovering full function & independence (71%) and good quality of life (83%), while only 6% of physicians expected independence and only 4% a good quality of life. But only 26% of surrogates recalled the doctorproviding any information about survival, quality of life, and need for caregiving. A third of the families reported it was the doctor's decision, not theirs, to perform tracheostomy. (The real outcomes at 1 year: 56% of those patients did survive; 9% regained independence; surrogates reported good quality of life in 33% of the survivors, or 19% of the original sample. The majority of survivors were cared for at home, and surrogates reported significant personal emotional and financial strain.
  • 66. 66 Question 33 of 82 (score?) You are using airway pressure release ventilation (APRV or "bi-level") for Ms. P, a 76-year-old woman with ARDS from sepsis. She has been on mechanical ventilation for 5 days. She is in shockand is on vasopressors. Her ventilator settings are: P[high] = 30; P[low] = 0 T[high] = 5 sec; T[low] = 0.6 sec PEEP = 0; FiO2 = 0.55 Her most recent arterial blood gases are: pH 7.25, paCO252; paO2 62 (8 hours ago) pH 7.19, paCO262; paO2 60 (30 minutes ago) Her ideal bodyweight is 60 kg and she is receiving tidal volumes (Vt) of 360 mL. Mean airway pressure is 24 mmHg. The bestthing to do next would be to: A. Increase FiO2 to 0.65 B. Increase PEEP to 5 C. Decrease T[high] to 4 sec D. Decrease P[high] to achieve Vt of 5mL / kg E. Decrease T[low] to 0.4 sec We feelthat C is the best answer. Ms P is hypoventilating due to an inadequate minute ventilation being delivered. Airway pressure release ventilation (APRV or "bi-level") cycles between two alternating levels of CPAP, P[high] and P[low], whose durations are determined by T[high] (usually several seconds)and T[low] (usually a fraction of a second). This pattern maintains a stable high mean airway pressure, keeping alveoli open and
  • 67. 67 augmenting oxygenation--an "openlung" strategy. APRV can also be viewed as a form of pressure-controlled, inverse-ratio ventilation. Ventilation (exhaled tidal volume and CO2) occurs during the pressure drop from P[high] to P[low] (hence "pressure release ventilation"). PEEP is usually set to 0, but the short time at P[low] creates beneficial auto-PEEP and prevents alveolar collapse. Patients may take small additional breaths throughout the entire cycle; these Vt improve alveolar recruitment and oxygenation but contribute little to minute ventilation. The most effective way to increase minute ventilation in APRV is to decrease T[high], which increases the frequency of breaths. Although Ms P's ABG is acceptable for a patient with ARDS, her acidemia is worsening. Many would argue that hypercarbic respiratory acidemia (down to pH of 7.15 or so)is well-tolerated and poses little risk. However, the trend is concerning and potentially dangerous should it continue. This is an easy ventilator change to make with no apparent risk. Reducing minute ventilation further by reducing delivered tidal volumes would be unwise when peak and mean pressures are acceptable, and while her respiratory acidemia is worsening. Increasing PEEP would also reduce her delivered minute ventilation, because there would be a smaller pressure gradient between P[high] and P[low]. Decreasing T[low] by 0.2 sec would probably have little effect. Her oxygenation is fine; changing FiO2 is not necessary. Question34 of 82 (score?) A 64 year old man complains of shortness of breath after a successfulcoronary artery bypass graft surgery 5 weeks prior. He is found to have a large left-sided pleural effusion (about 50% of the hemithorax) and readmitted to his surgeon's team. A thoracentesis drains 1,800 mL of fluid, relieving his dyspnea, A follow-up chest film shows minimal remaining fluid. He had no other presenting symptoms besides dyspnea, and is asymptomatic now. The fluid was exudative (LDH 400) and had 1,100 white cells/mL, 71% lymphocytes. He looks and feels better and wants to go home. The cardiothoracic surgeon asks you your opinion. What's the best advice to give?
  • 68. 68 A. Stay in the hospital to monitor for reaccumulation and for culture results. B. Dischargehome; if the fluid reaccumulates we'll justtap it again. C. Dischargehome; if the fluid reaccumulates we'll place a chest tube. D. Dischargehome; if the fluid reaccumulates we'll place a pleural drain. E. Place a chesttube now, as the likelihood of rapid recurrence is high. You answeredB. We agree that B is the best answer. Pleural effusions are common after cardiac surgery. Most are due to inflammation and anatomic disruption from the surgery itself, and are generally benign. These are called nonspecific pleural effusions, and may occurearly (<30 days) or late (>30 days) after surgery. If small, asymptomatic, and developing early after surgery, nonspecific pleural effusions require no further evaluation. Late, large (>25% of the hemithorax) and/or symptomatic nonspecific pleural effusions should be drained and the fluid analyzed in the usual fashion, as hemothorax, parapneumonic effusions, chylothorax, and other etiologies are also possible. The fluid in both early and late nonspecific pleural effusions is usually exudative and leukocyte-rich, reflecting the causative inflammatory process. Early effusions are often bloodyand rich in eosinophils and neutrophils; late effusions are more clear (but exudative) and rich in lymphocytes. Treatment of nonspecific pleural effusions (if needed) post-cardiac surgery is drainage with thoracentesis. More aggressive measures are rarely required, although repeat thoracenteses (up to 3) may be necessary. NSAIDs and glucocorticoids have also been used in a non-evidence based fashion. Postpericardiotomysyndrome is a distinct, persistent inflammatory state that occurs days to weeks after cardiac surgery, and frequently results in pleural effusions. In contradistinction to nonspecific pleural effusions, pericardial effusions, fever, chest pain, leukocytosis, pleural and/or pericardial rub are usually present (similar to / forme fruste of Dressler's syndrome). The pleural effusion may be small, but is often bloody, exudative, and leukocyte-rich. NSAIDs and/or glucocorticoids are the mainstay of treatment.
  • 69. 69 Post-CABG effusions that recur and persist after multiple thoracenteses, or persistent effusions discovered late (after months have passed)may require decortication to prevent trapped lung and loss of lung function. Question35 of 82 (score?) A 51-year-old woman comes to your clinic for follow-up. She has coughed up colored sputum a few days a week for the past 15 years, very infrequently streaked with blood (none lately). It may be getting a little worse, slowly. She feels at her baseline today. A chest X-ray and high-resolution CT scan show extensive bronchiectasis in the lower lobes bilaterally. She has diabetes, a history of repeated respiratory illnesses, and a history of non-Hodgkin lymphoma in remission.She was previously followed by a pulmonologist and says she was told she does not have asthma. She has three biological children. She never smoked. Other test results include:  HIV antibody and TB skin test are negative.  Total IgElevels are normal.  Sweat chloride test is normal.  White cell count is 11 with no eosinophils. She had a bronchoscopylast year during an exacerbation, showing purulent secretions. The BAL grew >100K H.influenzaeand she was treated with antibiotics with a return to her baseline. What would be the best next test to order to determine the underlying cause of her bronchiectasis? A. CFTR mutation testing B. Quantitative immunoglobulins C. IgG subclass levels D. Precipitating serumantibodies to Aspergillus E. Biopsy of nasalepithelium We feelthat A and B are the best answers.
  • 70. 70 Because some causes of bronchiectasis are treatable, finding the etiology (when possible) is important. (Even after extensive workup, as many as 50% of cases of bronchiectasis have no identifiable cause, though.) This woman has features suggestive of chronic variable immune deficiency (CVID), in which impaired B-cell differentiation leads to low immunoglobulin production. Levels of IgG, and usually IgA and/or IgM are all below the lower limit of normal. An inadequate specific IgG responseafter intentional vaccination (e.g., with tetanus & diphtheria booster)can help make the diagnosis. Most CVID patients have a history of bacterial pneumonia and/or rhinosinusitis (esp. Pneumococcus, Mycoplasma, and Hemophilus); chronic lung disease and bronchiectasis are common but their exact prevalences are unknown. Gastrointestinal disease, autoimmune disease (especially hematologic, e.g. immune thrombocytopenic purpura), and malignancy (especially non-Hodgkin lymphoma and gastric cancers), are also common. Noncaseating granulomas in the liver, lungs, and lymph tissue can occurin CVID, mimicking sarcoid. CVID is in part a diagnosis of exclusion, and other immune deficient states must be ruled out. Certain heterozygous CFTR mutations can cause diffuse bronchiectasis in people with negative sweat chloride testing and without cystic fibrosis. Measurement of IgG subclasses should only be performed after first checking total immunoglobulin levels. Information provided in the question stem makes most of the other major known causes of bronchiectasis less likely: Condition Features Diagnosis Primary ciliary dyskinesia Situs inversus in 50%; Infertility in most men & >50% of women Exhaled nasalnitric oxide; Nasal brush biopsy Cystic fibrosis 7% are diagnosed after age 18; Pseudomonas, non-TB mycobacteria; Often upper lobe bronchiectasis Sweat chloride test >30; mutation screening of CFTR gene Allergic Asthma; usually (+)blood Elevated total IgEand
  • 71. 71 bronchopulmonary aspergillosis eosinophils (+)IgE& IgG to Aspergillus; central bronchiectasis; skin test reactive to Aspergillus Common variable immunodeficiency Pneumococcus, Hemophilus, Mycoplasma infections; see above Low levels of IgG and either IgA or IgM; impaired responseto vaccines Airway obstruction Impaired consciousness (seizures, strokes, EtOH) CT scan; bronchoscopy Immunedeficiency Organ transplant, HIV History Young's syndrome Sinusitis, azoospermia Exclusion Alpha-1 antitrypsin deficiency Less common alternative presentation fromemphysema A1ATlevel Non-TB mycobacteria Can be primary process (esp. in nonsmoking women age 50+) or secondary infection in existing bronchiectasis Sputumculture x 2, or bronchoscopic samplex 1, with radiologic/clinical compatibility Question41 of 82 (score?) A 70 year old man had a right upper lobectomy for lung adenocarcinoma 6 months ago. He came today to his oncologist for a routine surveillance CT scan, which is
  • 72. 72 shown below. The air-fluid level was somewhat lower on his previous scan 3 months ago. No abnormal fistulous tracts are seen on the remainder of the CT. His white blood cell count is 9 today. He has "good days and bad" and today is one of his average days. He went for his usual half-mile walk this morning and drove himself in. His vital signs are normal. His chronic cough productive of white sputum is unchanged. The oncologist, your colleague in a multispecialty practice, asks you what you think. The bestnext step is:
  • 73. 73 A. Observation B. 14 day courseof Augmentin; call if fever develops C. Intravenous broad-spectrumantibiotics D. Needle aspiration of fluid E. Bronchoscopy with lavage We feelthat A is the best answer. The surgically created cavity left behind after a lobectomy or pneumonectomy will normally slowly fill up with sterile serous fluid, eventually forming an opacity on imaging with mediastinal shift toward the surgical side. This is likely part of that process, although it usually happens in weeks, not months. The late appearance of fluid justifies consideration that a new infection is responsible. However, in the absence of signs or symptoms, the images shouldn't by themselves dictate workup or treatment. A bronchopleural fistula could be considered (the delayed or chronic form; most BPFs occurin the early postop period), but would always be associated with symptoms (fever and cough, nonspecific malaise). Acute/subacute BPFs usually cause a new relative mediastinal shift to the contralateral side; delayed BPFs may not, as fibrosis (anchoring things in place) has usually occurred. CT scanning is not sufficiently sensitive for detecting BPF, and absenceof a visible tract on CT does not rule out the condition. This man went home after his visit and did fine. Fleischner Society Recommendations Incidental Pulmonary Nodule Follow-up
  • 74. 74 Question43 of 82 (score?) A 46 year old man comes to your clinic for management of his asthma. He takes high-dose inhaled corticosteroids and a long-acting beta agonist, along with a leukotriene inhibitor. His adherence and technique are perfect. He still has symptoms of cough, wheezing, and chest tightness that bother him most days and nights each week. He is using albuterol daily. The symptoms persist when he goes on vacation out of state. Sputum culture is negative. IgE level is 3,600 ng/mL. His primary doctorobtained imaging and a chest CT, which are shown.
  • 75. 75
  • 76. 76
  • 77. 77